Test task in clinical pharmacology. Pre-exam test in pharmacology

1.What is the name of the branch of pharmacology that studies the absorption, distribution, biotransformation and excretion of drugs?

Pharmacokinetics.

Pharmacodynamics.

2.What is the name of the branch of pharmacology that studies the types of action of drugs, pharmacological effects, and mechanism of action?

Pharmacodynamics.

Pharmacokinetics.

3. The main mechanism of drug absorption in the gastrointestinal tract:

Active transport.

Facilitated diffusion.

Passive diffusion through cell membranes.

Pinocytosis.

4. The main site of drug absorption is weak bases:

Small intestine.

5. The main site of drug absorption is weak acids:

Small intestine.

6.Which method of drug administration ensures 100% bioavailability?

Intramuscular.

Rectal.

Intravenous.

Through the mouth.

7. How will the absorption of drugs – weak acids – change when the acidity of gastric juice decreases?

Will increase.

Will decrease.

8. How will the absorption of drugs – weak bases – change when the acidity of gastric juice decreases?

Will increase.

Will decrease.

9. Substances are easily transported through passive diffusion through biological membranes:

Lipophilic.

Polar.

Hydrophilic.

10.Enteral route of drug administration:

Intramuscular.

Inhalation.

Sublingual.

Intravenous.

11. Parenteral route of drug administration:

Through the mouth.

Into the rectum.

Subcutaneous.

Sublingual.

12.Where do the absorption of most drugs take place?

In the oral cavity.

In the stomach.

In the small intestine.

In the large intestine.

13. The following can be administered intravenously:

Oil solutions.

Insoluble compounds.

Osmotically active compounds.

Microcrystalline suspensions.

Insoluble compounds.

14.What functional change in the body is caused by cardiac glycosides in heart failure?

Excitation.

Oppression.

Toning.

Calm.

15.What functional change in the body is caused by a drug that lowers blood pressure in arterial hypertension?

Excitation.

Oppression.

Toning.

Calm.

16.What is the accumulation of a drug in the body during repeated administration called?

Functional cumulation.

Sensitization.

Material cumulation.

Tachyphylaxis.

17. Tolerance is:

An allergic reaction of the body to repeated administration of the drug.

Reducing the pharmacological effect of repeated administration of the drug.

An irresistible urge to take medication again.

18. A decrease in the effect when administering drugs at short intervals is:

Tachyphylaxis.

Idiosyncrasy.

Sensitization.

Addiction.

19.Side effect that may occur only with repeated administration of drugs:

Idiosyncrasy.

Teratogenic effect.

Mutagenic effect.

Addiction.

20.Side effect that may occur only when using psychotropic drugs:

Idiosyncrasy.

Addiction.

Addiction.

Sensitization.

21. Determine the type of drug interaction: a patient with muscarine poisoning underwent gastric lavage with a suspension of activated carbon:

Summed up synergy.

Chemical antagonism.

Competitive antagonism.

Physical antagonism.

22. Mutagenic effect is:

23. Teratogenic effect is:

Damage to the genetic apparatus of the germ cell.

Impaired differentiation of fetal tissues, causing various anomalies.

A side effect that occurs in the first 12 weeks after fertilization and causes the death of the embryo.

24. Embryotoxic effect is:

Damage to the genetic apparatus of the germ cell.

Impaired differentiation of fetal tissues, causing various anomalies.

A side effect that occurs in the first 12 weeks after fertilization and causes the death of the embryo.

MODULE 3 CLINICAL PHARMACODYNAMICS. PATIENT AND MEDICINE

MODULE 3 CLINICAL PHARMACODYNAMICS. PATIENT AND MEDICINE

After mastering the topic, the student should know

1. Definition of pharmacodynamics.

2. Definitions of antagonists, agonists, partial agonists.

3. Types of drug target molecules (receptors, enzymes, ion channels).

4. Types of pharmacological response: expected pharmacological response, hyperreactivity, tachyphylaxis, idiosyncrasy.

5. Principles for developing programs for monitoring the effectiveness of medicines.

6. Acute pharmacological test (concept, purposes, indications, rules of conduct).

7. Methods for assessing the effect of drugs on quality of life.

8. Methodology for collecting pharmacological history (concept, clinical significance, collection rules, interpretation).

9. Patient adherence to treatment - compliance (concept, factors influencing adherence to treatment, methods of increasing patient adherence to treatment).

10. Responsible self-medication.

After mastering the topic, the student should be able to

1. Interpret information on the pharmacodynamics of drugs (from the instructions and TKFS) for an individualized choice of drugs.

2. Develop a program for monitoring the effectiveness of drugs, taking into account their pharmacological effects.

3. Assess the impact of medications on quality of life.

4. Conduct an acute pharmacological test and interpret its results to select medications.

5. Collect and interpret pharmacological history.

6. Carry out activities that increase the patient’s adherence to drug treatment.

Literature required to master the topic

Main

Kukes V.G. Clinical pharmacology. - M.: GEOTAR-Media, 2008. - P. 80-94, 95-117.

Additional

Novik A.A., Ionova T.I. Guidelines for quality of life research in medicine. - M.: Olma media group, 2007. - 320 p.

Sergeev P.V., Shimanovsky N.L., Petrov V.I. Receptors. - Volgograd, 1999. - 640 p.

Lecture you need to listen to to master the topic

Kukes V.G.“Introduction to clinical pharmacology. Pharmacodynamics".

http://lech.mma.ru/clinpharm/ucheb/pharm/lekt/1

Complete test tasks for self-control

Select one or more correct answers.

I. Pharmacodynamics includes:

A. Processes of absorption, distribution, metabolism and excretion of drugs.

B. Mechanism of action, localization of action and types of action of drugs.

B. Movement of drugs in the patient’s body. D. Physico-chemical properties of drugs.

II. Match the terms and their definitions:

A. Agonists.

B. Antagonists.

B. Partial agonists.

1. Drugs that bind to the same receptors as endogenous mediators, causing a “zero effect.”

2. Drugs that bind to the same receptors as endogenous mediators, causing an effect equal to or greater than the effect of this mediator.

3. Drugs that bind to the same receptors as endogenous mediators, causing an effect greater than zero, but less than the effect of this mediator. III. Match the terms and their definitions:

A. Idiosyncrasy. B. Tachyphylaxis.

B. Tolerance.

1. A decrease in the therapeutic effect observed with repeated use of drugs.

2. A genetically determined perverse reaction to a certain drug, manifested by increased sensitivity to it and/or a long-term effect and associated with a genetically determined defect in enzyme systems.

3. A decrease in the therapeutic effect observed with long-term (repeated) use of drugs.

Task 3.1. Give examples of drugs - agonists and antagonists of various types (or rather classes) of receptors by filling out the diagram in Fig. 3.1. In the center, in the rectangles, indicate the receptors; above and below in the arrows are examples of drugs that are their agonists or antagonists.

More detailed information about the interaction of drugs with target molecules in the lecture by R. Leurs “Drug-receptor interaction” (in English): www.ux1.eiu.edu/~cfthb/classes/4790/pdfs/Drug-Receptor_Interactions.pdf

Why does a doctor need information about the pharmacodynamics of drugs and how to interpret it

Pharmacodynamics is a branch of clinical pharmacology that studies the mechanism of action, localization of action and types of action of drugs in a patient. In a simplified form, pharmacodynamics allows you to understand what a drug does to the patient’s body. In this case, the main or therapeutic effect of the drug is distinguished - pharmacological

Rice. 3.1. Medicines - agonists and antagonists of various receptors

an effect that is aimed at achieving the treatment goals. All pharmacological effects that are not related to the main one are regarded as side effects, which can be not only “negative” for the patient’s health (unwanted side reactions), but also “positive”. Information on the pharmacodynamics of drugs is contained in the “Pharmacological action” section of the instructions and TKFS and is intended for the doctor. Based on this information, the doctor forms an idea of ​​the significance of pharmacodynamic effects in the development of therapeutic and adverse drug reactions (Table 3.1), which determines the indications and contraindications for the use of the drug, and, consequently, the choice of this drug. Thus, the doctor must understand the clinical significance of the information presented for the individualized choice of both a group of drugs and individual drugs (including taking into account the results of an acute pharmacological test), the development of a program for monitoring the effectiveness of treatment (including assessing the dynamics of the patient’s quality of life during the use of LS).

Table 3.1. The importance of information on the pharmacodynamics of bisoprolol for understanding its therapeutic benefits

and adverse reactions

Medicine

Target molecule and nature of interaction with it

(agonist or antagonist)

Localization of action

Types of action

Pharmacodynamic effect

Therapeutic effect and corresponding indication for use

Adverse adverse reaction

Bisoprolol

β1-adrenoreceptors

Cardiomyocytes

Negative ino-, dromo-, chrono-, bathmotropic. Reduced myocardial oxygen demand.

Hypotensive

Antianginal - IHD. Decrease in heart rate, antiarrhythmic effect - tachyarrhythmias; CHF.

Hypotensive -

arterial

hypertension

Bradycardia. Depression of AV conduction

In large

β 2 -adrenoreceptor-

Pancreas, skeletal muscles, smooth muscles of peripheral arteries, bronchi and uterus

Decreased bronchial tone; hyperglycemic effect; decreased peripheral circulation

Not used

Bronchospasm Impaired carbohydrate metabolism and peripheral blood flow

Complete the assignment for independent work

Task 3.2. Describe the pharmacodynamics of the drug proposed by the teacher or drugs in your future specialty by filling out the table by analogy with table. 3.1. Use the “Pharmacological action” section of the TKFS.

How does knowledge of pharmacodynamics help a doctor choose a specific drug from a group?

Knowledge of the characteristics of the pharmacodynamics of a drug underlies the selection of a specific drug within a selected group in accordance with the principle of matching the profile of the drug (features of the pharmacodynamics of the drug from the section “Pharmacological action” of the instructions and TKFS) and the patient’s profile (individual characteristics of the patient). As an example, we present the choice of drugs from the group of β-blockers for the treatment of arterial hypertension based on pharmacodynamics (Table 3.2).

Table 3.2. Selection of a drug from the group of β-blockers for the treatment of arterial hypertension based on pharmacodynamics

Features of pharmacodynamics

Selection principles

Propranolol Carvedilol

β Γ blockade

and β2-adrenergic receptors

We choose for concomitant portal hypertension. We do not choose for concomitant COPD, diabetes mellitus, hyperlipidemia, in young and middle-aged patients (sexually active)

Atenolol Metoprolol Bisoprolol Nebivolol

Selective blockade of βl-adrenergic receptors

We choose for concomitant COPD (especially bisoprolol and nebivolol), in young and middle-aged (sexually active) patients (especially bisoprolol and nebivolol), diabetes mellitus in the compensation stage (with well-selected hypoglycemic therapy)

End of table. 3.2

Features of pharmacodynamics

Selection principles

Carvedilol Nebivolol

The presence of additional vasodilating properties: in nebivolol due to the ability to stimulate the release of nitric oxide in the vascular endothelium, and in carvedilol due to the α-adrenergic blocking effect

Select for concomitant obliterating atherosclerosis of the vessels of the lower extremities (except in cases of critical ischemia)

Carvedilol

Metoprolol

(in the form of bitches-

Bisoprolol

Nebivolol

Proven in randomized studies is a positive effect on the prognosis of patients with chronic heart failure due to a decrease in the activity of the sympathetic adrenal system

We choose it for concomitant chronic heart failure in the compensation stage (in the absence of signs of fluid retention). At the same time, we choose nebivolol only for patients with chronic heart failure over 75 years of age (there is evidence of the effectiveness of this drug only for this category of patients). Preference should be given to carvedilol in cases of combination of chronic heart failure and stage III arterial hypertension (carvedilol has a more pronounced antihypertensive effect due to the α-adrenergic blocking effect)

Complete the assignment for independent work

Task 3.3. Using the “Pharmacological action” section of the TKFS, develop principles for choosing drugs within a group of drugs on the topic of a practical lesson (the group of drugs is proposed by the teacher)

or from your future specialty. To do this, fill out the table by analogy with table. 3.2.

The choice of drugs from a group should be made not only on the basis of pharmacodynamics, but also pharmacokinetics, the consideration of which also contributes to the choice of an individualized dosage regimen (see module 2).

How to develop an individual program for monitoring the effectiveness of a drug?

In the process of using drugs, the doctor must monitor the effectiveness, i.e. the degree of achievement of the treatment goals (primary and secondary). To do this, it is necessary to develop an effectiveness monitoring program, which will represent methods (clinical, laboratory and instrumental) that evaluate the development of the therapeutic effects of drugs, as well as the timing of their implementation and the level of change in the results of these methods during treatment (Table 3.3). The selected methods in the program for monitoring the effectiveness of the drug should provide information to the doctor about the degree to which the patient’s primary and secondary treatment goals are achieved, depending on the indications for the use of the drug. Based on the results obtained, the doctor makes appropriate decisions: adjusts the dose of the drug, continues the use of the drug, cancels the drug, adds other drugs. The physician also makes these decisions in accordance with the results of the safety monitoring program (see Module 4). Efficiency and safety monitoring programs are implemented in parallel and complement each other. To develop a program for monitoring the effectiveness of drugs, you should use information from the sections “Pharmacological action” and “Indications” of the instructions and TKFS.

Complete the assignment for independent work

Task 3.4. Develop a program for monitoring the effectiveness of the use of drugs proposed by the teacher, or by independently choosing drugs in your future specialty. Fill out the table by analogy with table. 3.3. Use the “Pharmacological action” section of the TKFS.

Table 3.3. Methods for monitoring the effectiveness of treatment for a patient with exacerbation of duodenal ulcer

Indication for this patient

Medicine

Methods for assessing effectiveness

Healing of ulcerative defect

Drug 1 - rabeprazole

Mechanism 1 . Inactivates sulfhydryl groups of H+/K+-ATPase. Blocks the final stage of HCl secretion, reducing the content of basal and stimulated secretion. Mechanism 2. Penetrates the parietal cells of the stomach and concentrates in them, exerting a cytoprotective effect and increasing the secretion of bicarbonate

Clinical methods: reducing or stopping pain, eliminating heartburn, belching. Laboratory methods: laboratory eradication markers N. pylory(when used together with antibiotics) - see below. Instrumental methods: EGDS - scarring of the ulcer, reducing its size. pH-metry - increasing intragastric pH

Eradication H. pylori

Drug 1 - rabeprazole

Mechanism 1. Increases the concentration of antibacterial agents in the gastric mucosa.

Mechanism 2. Increases pH, increasing the activity of antibiotics (amoxicillin, clarithromycin). Mechanism 3. Anti-Helicobacter effect (suppress growth H. pylori in vivo, acting on the bacterial ATPase system)

Clinical methods: none. Laboratory methods: urease breath test 3-4 weeks after the end of therapy.

Instrumental methods:

histological examination of the biopsy;

urease test with biopsy

End of Table 3.3

Indication for this patient

Medicine

Mechanism of action of the drug corresponding to the indication

Methods for assessing effectiveness

Clarithromycin

Mechanism 1. Suppresses protein synthesis by reversibly binding to the 50S ribosomal subunit H. pylori, providing a bacteriostatic effect

Amoxicillin

Mechanism 1. They disrupt the formation of bacterial cell walls due to irreversible binding to penicillin-binding proteins, providing a bactericidal effect

How to use quality of life assessment as a clinical method for monitoring the effectiveness of a drug?

An important clinical method for monitoring the effectiveness of drugs is the assessment of quality of life over time. Quality of life is an integral indicator that includes several components:

Functional state (performance, exercise tolerance, doing homework);

Symptoms associated with the disease or its treatment (pain, shortness of breath, side effects of drugs);

Mental state (depression or agitation, which can be either a consequence of the disease itself or the use of drugs);

Social activity;

Sexual function;

Satisfaction with medical care.

Factors that worsen the quality of life can be divided into two groups:

Factors associated with the disease (symptoms of the disease that interfere with life);

Factors associated with drug therapy (inconvenience of drug use, development of adverse drug reactions).

The severity of factors associated with diseases is assessed using questionnaires. At the same time, there are universal questionnaires for assessing the quality of life in patients with any disease, but they are usually very cumbersome, filling out and interpreting them takes a lot of time for patients and doctors, so special questionnaires have been developed for patients with the most common diseases. The patient must independently answer the questionnaire, and the doctor calculates the total number of points in accordance with the patient’s answers. The dynamics of the sum of points during drug treatment will be a method for monitoring the effectiveness of the drug.

Universal quality of life questionnaire EQ-5D (in Russian): http://forum.disser.ru

More detailed information on assessing quality of life can be found in the article: Myasoedova N.A., Tkhostova E.B., Belousov Yu.B. On assessing the quality of life in various cardiovascular diseases: http://www.trimm.ru/php/content.php?group=2&id=3726

Analysis of factors associated with drug therapy allows for correction of drug use during treatment.

For example, let's analyze the quality of life of a 53-year-old patient diagnosed with coronary artery disease; post-infarction cardiosclerosis. Arterial hypertension of the 2nd degree, stage III, very high risk. CHF ΙΙ FC according to NYHA. The patient receives enalapril 5 mg 2 times a day, bisoprolol 2.5 mg 1 time a day, furosemide 40 mg 2 times a week, spironolactone 25 mg 1 time a day. The patient complains of shortness of breath when walking 300-500 m, climbing to the 4th floor (he lives on the 5th floor in a building without an elevator), night shortness of breath occurs approximately once a week; inability to engage in physical labor and sports

(previously worked as a car mechanic, was fond of skiing); on the day of taking furosemide, he is forced to stay at home due to the diuretic effect, so he is forced to switch to a less paid job with a shift schedule; also notes a slight dry cough (he refused to prescribe angiotensin receptor blockers due to their higher cost). Factors that worsen the patient’s quality of life are presented in Table. 3.4.

Table 3.4. Factors that worsen the quality of life of a patient with chronic

heart failure

The assessment of factors that worsen the quality of life associated with diseases in a given patient is carried out using a special questionnaire for patients with chronic heart failure, the so-called Minnesota Questionnaire (Table 3.5). A decrease in the score on this questionnaire during treatment with the listed drugs will indicate the effectiveness of therapy. The impact on the quality of life of the identified factors associated with drug treatment, in this case, can be reduced by adjusting the therapy:

Replace taking furosemide 2 times a week with daily use of hydrochlorothiazide at a dose of 25 mg per day;

Replace the ACE inhibitor enalapril with the angiotensin receptor antagonist losartan, which causes a dry cough much less frequently, but is similar in effectiveness;

Prescribe a combination drug containing losartan and hydrochlorothiazide in one tablet in appropriate doses.

Table 3.5. Minnesota Questionnaire for assessing the quality of life of patients with chronic heart failure

Has your heart failure prevented you from living as well as you would have liked during the past month because of:

1. Swelling of the legs and feet

0, 1, 2, 3, 4, 5

2. The need to rest during the day

0, 1, 2, 3, 4, 5

3. Difficulty climbing stairs

0, 1, 2, 3, 4, 5

4. Difficulty doing housework

0, 1, 2, 3, 4, 5

5. Difficulty traveling away from home

0, 1, 2, 3, 4, 5

6. Night sleep disturbances

0, 1, 2, 3, 4, 5

7. Difficulty communicating with friends

0, 1, 2, 3, 4, 5

8. Decrease in earnings

0, 1, 2, 3, 4, 5

9. Inability to play sports or hobbies

0, 1, 2, 3, 4, 5

10. Sexual violations

0, 1, 2, 3, 4, 5

11. Diet restrictions

0, 1, 2, 3, 4, 5

12. Feeling short of breath

0, 1, 2, 3, 4, 5

13. Having to stay in the hospital

0, 1, 2, 3, 4, 5

14. Feelings of weakness, lethargy

0, 1, 2, 3, 4, 5

15. Having to pay

0, 1, 2, 3, 4, 5

16. Side effects of medications

0, 1, 2, 3, 4, 5

17. Feelings like a burden to relatives

0, 1, 2, 3, 4, 5

18. Feelings of loss of control

0, 1, 2, 3, 4, 5

19. Feelings of restlessness

0, 1, 2, 3, 4, 5

20. Deterioration of attention, memory

0, 1, 2, 3, 4, 5

21. Feelings of depression

0, 1, 2, 3, 4, 5

Possible answers: 0 - no; 1 - very little... 5 - very much (the highest quality of life - 0 points; the lowest - 105 points).

Complete the assignment for independent work

Task 3.5. List the factors that determine the quality of life of the supervised patient by filling out a table similar to Table. 3.5.

Complete the assignment for independent work

Task 3.6. Find or develop your own questionnaire to assess the quality of life of patients with a disease in your future specialty. Use as an example a questionnaire to assess the quality of life of patients with chronic heart failure (Minnesota Questionnaire; see Table 3.5).

What is an acute pharmacological test and how can its results be interpreted for individualized drug selection?

An acute pharmacological test is an analysis of a single use of a drug in a supervised patient, which allows one to predict the therapeutic effect and the development of adverse drug reactions when using the drug.

Diagnostic pharmacological tests are widely used in clinical practice: a test with dobutamine to detect myocardial ischemia, a test with ACTH and dexamethasone when diagnosing adrenal insufficiency, a test with neostigmine when diagnosing myasthenia gravis, a diazepam test in the diagnosis of affective psychoses, a test with a bronchodilator when conducting spirometry. In some cases, a pharmacological test helps not only confirm the diagnosis, but also determine the potential effectiveness of treatment; for example, it is obvious that with a positive test with salbutamol, bronchodilators from the group of β2-adrenergic agonists will be effective.

The concept of an acute pharmacological test implies an assessment of the effectiveness and safety of a drug for a known diagnosis. In this case, selected methods for monitoring effectiveness and safety are used. To conduct an acute pharmacological test, the doctor needs information about the pharmacodynamics of the drug from the section “Pharmacological action” and about the pharmacokinetics of the drug from the corresponding section of the instructions for medical use

LS or TKFS.

Let's look at the rules for conducting an acute pharmacological test using an example. A general practitioner is called for a consultation in the neurology department with a 70-year-old patient with a diagnosis of cerebral atherosclerosis. Atherosclerotic stenosis of the internal carotid arteries. Stage II dyscirculatory encephalopathy due to an increase in blood pressure to 175/100 mm Hg, accompanied by

expecting a moderate headache. From the anamnesis it is known that the patient had an increase in blood pressure to 160/100 mm Hg twice over the last 7 years, the usual blood pressure is 140/80, antihypertensive therapy has not been previously administered. Initial heart rate - 86 per minute. How should antihypertensive therapy be selected using an acute pharmacological test? In this case, the patient has a high risk of developing cerebrovascular accident with an excessive decrease in blood pressure, therefore, the doctor requires enhanced monitoring of the safety of the therapy, and an acute pharmacological test is appropriate.

The patient was prescribed bisoprolol at a dose of 2.5 mg.

Blood pressure and heart rate were monitored:

After 1 hour - 170/95 mm Hg. st; 70 per minute;

After 2 hours - 160/90 mmHg; 68 per minute;

After 3 hours - 140/90 mmHg; 66 per minute;

After 4 hours - 110/70 mm Hg; 55 per minute;

After 6 hours - 115/70 mm Hg; 57 per minute;

After 12 hours - 160/70 mmHg; 58 per min.

According to TKFS, the maximum concentration of bisoprolol is achieved 2-4 hours after administration. In this case, after 4 and 6 hours the patient experienced an excessive decrease in blood pressure, and after 12 hours the therapy was insufficient. Therefore, the initial dose of bisoprolol should be reduced, and after 8-10 hours an additional dose of the drug should be taken.

What is a pharmacological history and how to collect it?

Collecting a pharmacological history is a mandatory procedure for a doctor. Pharmacological history is a set of information about the medications previously taken by the patient, methods of their use, doses, effectiveness, unwanted side reactions, signs of intolerance, drug dependence. Pharmacological history allows us to identify previously effective drugs and/or drugs that caused the development of adverse drug reactions. This may determine the choice of both groups and specific drugs and their doses. To do this, the doctor needs:

Identify which drugs the patient cannot tolerate;

Determine how drug intolerance specifically manifested itself;

It is necessary to pay attention that patients often interpret the doctor’s questions in their own way, for example, by an allergy to nitroglycerin, many patients understand a headache;

Determine the cause of intolerance:

The true allergic reaction and its severity. For example, in case of a mild allergic reaction to penicillins (urticaria), it is possible to prescribe β-lactam antibiotics from the cephalosporin group, and in case of angioedema or anaphylactic shock, all drugs from this class are contraindicated;

The development of severe adverse reactions, in this case, it is necessary to determine the correct dosage regimen and administration of the drug, and take into account possible interactions. An example is glycoside intoxication when taking digoxin at a daily dose of 0.25 mg together with furosemide;

The development of non-severe adverse reactions that affect the patient’s quality of life, for example, cough when taking ACE inhibitors;

Incorrect dosage and/or medication regimen. For example, too high a dose of an antihypertensive drug can lead to orthostatic collapse; chewing retarded forms of nifedipine may be accompanied by severe tachycardia, weakness, and hot flashes;

Assess the effectiveness of previous therapy and identify possible reasons for ineffectiveness:

Irregular use of the drug (you should pay attention to the fact that some patients, when they say that they take the medicine constantly, mean when they feel unwell);

Insufficient dose/frequency of drug administration. For example, taking amoxicillin/clavulanic acid at a dose of 625/125 mg 2 times a day does not provide the necessary antibacterial effect;

Incorrect drug administration technique. For example, inhaled bronchodilators may be ineffective if the drug is not inhaled correctly; therefore, instead of increasing the dose, it is necessary to educate the patient;

Development of tolerance. When tolerance to nitrates develops, it is necessary to ensure a “nitrate-free interval” and not increase the dose of the drug;

Interactions with other drugs, food, nutritional supplements, etc. For example, the use of clopidogrel together with proton pump inhibitors reduces the antiaggregation effect of the former;

Severe course of the disease. The ineffectiveness of nitrates may indicate the development of unstable angina;

Wrong diagnosis. For example, nitroglycerin is almost ineffective for cardialgia of non-coronary origin;

Withdrawal syndrome. A hypertensive crisis resulting from the withdrawal of clonidine, as a rule, is not controlled by standard means and requires the use of clonidine.

Sometimes collecting a pharmacological history helps clarify the diagnosis. For example, interstitial pulmonary fibrosis can be caused by amiodarone. Pharmacological history is of particular importance during exacerbations of chronic diseases, for example, hypoglycemic coma can be caused by a relative or absolute overdose of hypoglycemic drugs.

Complete the assignment for independent work

Task 3.7. Formulate questions to collect a pharmacological history that corresponds to the necessary information in a patient with a disease in your future specialty, in another clinical situation suggested by the teacher, or in a supervised patient. Fill in the table. 3.6.

Table 3.6. Pharmacological history

End of table. 3.6

Information required for pharmacological history

Formulated question

Information received from the patient

The effectiveness of currently used drugs

Tolerability (safety) of currently used drugs

Medicines previously used in similar situations

Reasons for stopping previously used medications

Other medicines used for concomitant diseases or for other purposes (oral contraceptives)

Concomitant therapy with “alternative” remedies: herbal remedies, homeopathic medicines

Adverse drug reactions when taking previously used medications

Attitudes towards alcohol, smoking, drugs

What is compliance (adherence), and why does a doctor need to improve it?

Compliance is the patient's adherence to treatment. The effectiveness and safety of medications prescribed to a patient may depend on compliance. No modern and effective treatment method will be effective enough if the patient does not

He understands why he needs it and does not follow the instructions. It should be remembered that the patient’s idea of ​​his illness and the necessary treatment is formed not only from the words of the doctor, but also from friends, the media, and advertising. It is often difficult to convince a patient to comply with medical recommendations. In addition, it must be taken into account that it is sometimes more difficult for a young doctor to influence the opinions of older patients in terms of age and social status. Below are some possible reasons for a patient’s non-compliance with medical recommendations: reluctance to take medications (afraid of “using chemistry”, adherence to traditional medicine, psychics, etc.):

The patient may be afraid of unwanted side effects (even previously unknown);

High cost of treatment;

Inconvenience associated with treatment (for example, the need to stop driving a car when taking tranquilizers, a pronounced diuretic effect when taking diuretics, the need for frequent monitoring of coagulograms when taking anticoagulants);

Taking a large number of tablets at the same time;

Duration of treatment, especially when the patient does not feel the onset of the effect (for example, many patients suffering from arterial hypertension often ask the following question: “Why should I take so many medications all my life if I don’t feel my blood pressure?”) or when an unwanted side reaction is more “ significant” for the patient than the therapeutic effect of the drug.

Complete the assignment for independent work

Task 3.8. Analyze a case from clinical practice. When answering questions, use information from the TKFS “Bisoprolol” (sections “Pharmacokinetics”, “Dosage regimen”) of the State Register of Medicines (www.regmed.ru).

Patient P., 82 years old, was prescribed bisoprolol at a dose of 5 mg once a day for arterial hypertension by her local therapist. At the next visit to the doctor, due to high blood pressure, it is recommended to increase the dose of bisoprolol to 10 mg per day. However, the pharmacy where the patient went did not have 5 mg bisoprolol tablets, so she purchased 2.5 mg tablets. The patient started taking bisoprolol 2.5 mg 4 times a day,

subsequently explaining this by the inconvenience of taking 4 tablets at once. After 5 days of taking bisoprolol in this regimen, the patient developed a picture of a hypertensive crisis, and therefore she was hospitalized in the hospital.

1. What is the most likely reason for the development of a hypertensive crisis in the patient, based on the pharmacokinetics of bisoprolol?

2. What are the reasons for the patient’s low compliance?

To ensure maximum compliance, before prescribing a drug to a patient, the doctor must:

Decide whether the patient will be able to comply with the treatment plan for objective reasons and whether it is possible to ensure regular monitoring of therapy (for example, visits to the clinic to monitor the INR when prescribing warfarin may be difficult for elderly patients, and elderly patients may forget to take medications, etc.) ;

If possible, prescribe medications 1-2 times a day;

If possible, prescribe combination drugs;

Prescribe convenient means of “delivery” of drugs (for example, inhalers);

Do not change one drug to another unnecessarily and do not confuse the patient with various drug names, use INN;

Provide clear and understandable application information

LP (Table 3.7).

Table 3.7. The information that the doctor must provide to the patient is

oral anticoagulant warfarin

Continuation of the table. 3.7

How and when to use the medicine

Warfarin is taken in the entire daily dose at one time, preferably at 5-7 p.m. The tablets are washed down with water. It is not recommended to take it with food, but can be taken on an empty stomach.

To avoid complications, the required dose of warfarin is monitored using a blood test called INR. This may sometimes be referred to as INR in laboratory responses. During the entire period of taking warfarin, the INR should be in the range of 2.0-3.0. If the INR is less than 2.0, then blood clotting is not reduced and thrombotic complications are possible. If the INR is more than 4.0, hemorrhagic complications are very real. An increase in INR from 2.5 to 4.0 indicates the need to reduce the dose of the drug, but usually does not pose a direct threat. For some diseases, the required upper limit of INR is 4.0-4.4.

If it is not possible to determine INR, monitoring by prothrombin time (PT) is acceptable, but this method is much less reliable. No other blood tests are needed to calculate your warfarin dose. To identify the side effects of the drug, a general blood test, urine test and some biochemical tests are periodically prescribed. Selection of the dose of warfarin.

The most difficult and responsible stage. The selection period on average takes from 1 to 2 weeks, but in some cases it increases to 2 months. During this time, you will need frequent INR determinations, up to 2-3 times a week or daily. Each time you receive another test result, your doctor will determine a change in the dose of the medication and the date of the next test. If in several tests in a row the INR remains in the range of 2.0-2.5, this means that the dose of warfarin has been adjusted. Further control of treatment will be much easier.

Continuation of the table. 3.7

Questions that need to be answered by the patient

Specific information for the patient regarding a specific drug

Warfarin dose control.

If the dose of the drug is selected, less frequent monitoring is sufficient - first once every 2 weeks, then once a month. The frequency of additional studies is determined separately. The need for an extraordinary determination of the INR may arise in a number of cases, which we will discuss below. If in any doubt, ask your doctor for advice

How long should the medicine be used?

Warfarin therapy is long-term, sometimes lifelong. In some cases, genetic studies are required to determine the duration of therapy

When and what positive effects of the drug can be expected

The drug is designed to protect your life, preventing the development of heart attacks, strokes, thromboembolism

Possible problems that may arise when using the drug, and what to do if these problems occur

The use of warfarin may cause bleeding.

Tell your doctor immediately if you find:

Black (tarry) stool color;

Pink or red urine;

Bleeding from the nose or gums (including when brushing your teeth);

Unusually heavy or prolonged bleeding during menstruation;

Bruises or swelling on the body that occur for no apparent reason;

Any significant changes in well-being and health;

The appearance of skin spots on the thighs, abdominal wall, mammary glands

Continuation of the table. 3.7

Questions that need to be answered by the patient

Specific information for the patient regarding a specific drug

What foods, drinks (including alcoholic ones) and other medicines (including herbal medicines) should be avoided?

What to avoid:

Use of other drugs that affect the blood system (including those containing acetylsalicylic acid);

The use of drugs that affect the metabolism and excretion of warfarin (antibiotics, antidiabetic drugs, etc.). If you need to take any new medicine, be sure to contact your doctor and tell him that you are taking warfarin;

Engaging in traumatic sports where blows, bruises, and falls are possible;

Intramuscular injections. In outpatient treatment, in most cases, medications can be selected for oral administration;

Repeatedly take the medicine within one day. If you do not remember whether you took warfarin today, skip the dose;

Diet changes.

Warfarin acts on blood clotting through vitamin K, which is found in varying amounts in food. There is no need to avoid foods high in vitamin K! Nutrition should be complete. You just need to make sure that there is no significant change in their proportion in the diet, for example, depending on the season. If you significantly increase your intake of foods rich in vitamin K while on a stable dose of warfarin, this may greatly weaken its effect and lead to thromboembolic complications. The maximum amount of vitamin K (3000-6000 mcg/kg) is found in dark green leafy vegetables and herbs (spinach, parsley, green cabbage), and in green tea up to 7000 mcg/kg; intermediate amount (1000-2000 mcg/kg) - in plants with pale leaves (white cabbage, lettuce, broccoli, Brussels sprouts). A significant amount of the vitamin is found in legumes, mayonnaise (due to vegetable oils), and green tea.

End of table. 3.7

Questions that need to be answered by the patient

Specific information for the patient regarding a specific drug

Fats and oils contain varying amounts of vitamin K (300-1000 mcg/kg), more of it in soybean, rapeseed, and olive oils. The vitamin K content in dairy, meat, bakery products, mushrooms, vegetables and fruits, black tea, and coffee is low (no more than 100 mcg/kg). Regular consumption of berries and cranberry juice may increase the effect of warfarin.

Small doses of alcohol with normal liver function do not affect anticoagulant therapy, but alcohol consumption must be treated with caution.

Taking multivitamins that contain vitamin K may reduce the effect of warfarin

What to do if you miss a dose of medication

Take the drug the next day

Where to get more detailed information about the drug

Instructions, TKFS State Register of Medicines

Complete the assignment for independent work

Task 3.9. In accordance with what is stated in table. 3.7 Plan out recommendations for the patient on the use of any drug. Use the instructions for medical use of drugs and TKFS.

In the USA, drug instructions for patients were developed by the National Institutes of Health and are freely available on the Internet: http://www.nlm.nih.gov/medlineplus/druginformation.ritml

Are there additional methods to improve compliance?

Effective methods for improving compliance are also considered:

Keeping self-observation diaries;

Development of an individual treatment plan for the patient;

Visiting schools of patients with various diseases. It is very important that the patient not only carefully

taking drugs, but could also adequately independently assess his condition, take the necessary measures and consult a doctor in a timely manner. For this, it is useful to keep a self-observation diary (Table 3.8), where the patient would record his complaints, key laboratory and instrumental indicators (blood pressure level for arterial hypertension, diuresis volume for heart failure, body temperature for an infectious disease, glucose level for diabetes mellitus, etc.) , as well as additional drug intake. Such a diary will allow the doctor to more adequately assess the dynamics of the condition and the effectiveness of the therapy, and will also increase the patient’s adherence to treatment.

Table 3.8. Self-observation diary of a patient with bronchial asthma

Complete the assignment for independent work

Task 3.10. Using table as an example. 3.8, develop a self-observation diary for a patient in the profile of your future specialty.

Another important component of improving compliance is the preparation of an individual treatment plan, where the patient will be given recommendations on in which cases it is necessary to independently change the medication intake, when to immediately consult a doctor and what to do in emergency situations (Table 3.9).

Table 3.9. Individual treatment plan for a patient with bronchial asthma

I. Your main treatment:

Take daily:

1. 25 mcg of salmeterol and 125 mcg of fluticasone, 2 puffs in the morning and evening.

2. Zafirlukast 10 mg in the morning and evening orally, 1 hour before meals or 2 hours after meals.

3. Before physical activity, take: salbutamol 0.2 mg 1-2 breaths over 15-30 minutes. If symptoms occur, take “on demand”: salbutamol 0.2 mg 1-2 puffs.

II. When is it necessary to increase treatment?

Answer the questions by analyzing your condition over the past week:

Symptoms appeared (shortness of breath, cough, whistling, chest tightness and

etc.) more than 2 times a day?

Has asthma kept you awake at night?

Have you ever had to use your on-demand inhaler more than twice? Has your physical activity decreased due to asthma?

Has the PSV level dropped below_l/min?

If you answered YES 3 times or more, you need to increase your treatment.

Take daily:

1. 25 mcg of salmeterol and 250 mcg of fluticasone, 2 puffs in the morning and evening.

2. Zafirlukast 20 mg in the morning and evening orally, 1 hour before meals or 2 hours after meals.

3. Assess your condition daily and adhere to this treatment regimen for a week.

III. When to see a doctor?

If there is no improvement within a week, make an appointment:

_(specify phone number)_(reception)

Doctor: Full name_

IV. Emergency situation.

You have severe shortness of breath and can only speak short sentences.

You have a severe asthma attack and are scared.

You use your on-demand inhaler every 4 hours and there is no improvement.

Take 2-4 breaths of salbutamol. Take 20 mg methylprednisolone orally. Contact “03” for help.

Continue inhaling salbutamol for 2-4 breaths every 20-30 minutes until the doctor arrives.

Complete the assignment for independent work

Task 3.11. Using the example below of an individual treatment plan for a patient with bronchial asthma (see Table 3.9 from Global Initiative for Asthma, GINA, 2006), develop an individual treatment plan for the patient in your future specialty.

Another way to increase compliance and motivation of patients for treatment is the creation of special health schools for patients with certain diseases (health school for patients with diabetes, arterial hypertension, bronchial asthma, chronic viral hepatitis, peptic ulcer, osteoporosis, myopia, etc.). Main tasks of schools:

1. Give the patient an idea of ​​his disease and tell him about modern treatment options.

2. Enable the patient to assess the severity of his condition and the adequacy of the treatment.

3. Teach the patient to independently recognize impending deterioration and prevent it.

4. Teach the patient self-help in the event of an exacerbation.

5. Explain the purpose of personal devices (for example, a spacer, peak flow meter, glucometer, tonometer, etc.).

6. Teach how to keep a health diary.

8. Develop in the patient the skills of “responsible self-medication”. Self-medication (as defined by WHO experts) is understood as “the reasonable use by the patient himself of drugs that are freely available for the purpose of preventing or treating minor health disorders before the provision of professional medical assistance.” The patient should be informed that “self-medication can only be used for a limited number of ailments” and should be limited in time. Among the drugs that can be used for self-medication, there should be only over-the-counter drugs used by patients strictly according to the instructions for use of the drug, i.e. package insert: indications, contraindications, doses, routes of administration, frequency of use, duration of use, etc.

An example of a lesson plan for a school for patients with type 2 diabetes mellitus, conducted at the endocrinology clinic of the Moscow Medical Academy. THEM. Sechenov (Table 3.10).

Table 3.10. Lesson plan for a school with type 2 diabetes mellitus Lesson 1. Topic “What is diabetes”

Basic concepts of diabetology (normal, high and low blood sugar levels, renal coma threshold). Treatment goals.

Symptoms of diabetes and the cause of their appearance.

“Four pillars” of complex therapy for type 2 diabetes. Lesson 2. Topic “Self-control”

What is included in the concept of self-control and why is it necessary?

Methods for self-monitoring of sugar in blood, urine, acetone in urine.

Training in practical skills of self-control.

Keeping a “Diary of a Type 2 Diabetes Patient.” Lessons 3, 4. Topic “Basic principles of diet therapy for type 2 diabetes”

The concept of the energy balance of a healthy person and a patient with

Type 2 diabetes.

The main components of food and the concept of calorie content. Ways to reduce the calorie content of the daily diet. Carbohydrates as the main source of energy, classification of carbohydrates. System of “bread units”, interchangeability of products containing carbohydrates. Food traffic light.

Consumption of sweeteners and sweeteners. What about alcohol? Lesson 5. Topic “Physical activity. Hypoglycemia"

Part I. Physical activity:

The need to expand physical activity in type 2 diabetes;

Basic rules for the dosage of physical activity;

Behavior before and during physical activity. Part II. Hypoglycemia:

What is hypoglycemia;

Causes of hypoglycemia;

Symptoms of mild and severe hypoglycemia;

What to do if symptoms of hypoglycemia occur.

End of table. 310

Complete the assignment for independent work

Task 3.12. Using the example given, develop a lesson plan for a patient's school in your future specialty.

Lesson 6. Topic “Late complications of diabetes: retinopathy, nephropathy, neuropathy”

Discussion of the most common complications of diabetes and the causes of their occurrence, introduction of the terms “retinopathy”, “nephropathy”, “neuropathy”.

Retinopathy: stages, symptoms, prevention, treatment. Nephropathy: stages, symptoms, prevention, treatment. Neuropathy: forms, symptoms, prevention, treatment. Lesson 7. Topic “Rules of foot care” Causes of leg damage in diabetes.

Prevention of leg injuries (do's and don'ts).

Training in shoe inspection skills and behavior when problems arise.

leg injuries. Lesson 8. Topic “Arterial hypertension, atherosclerosis and type 2 diabetes”

Part I. Arterial hypertension:

What is blood pressure;

Normal blood pressure levels, causes and mechanisms of blood pressure increase;

Symptoms of high blood pressure;

Dangers of high blood pressure;

Principles of modern antihypertensive therapy. Part II. Atherosclerosis:

What is atherosclerosis and the mechanisms of its development;

Normal blood cholesterol levels;

Manifestations of atherosclerosis;

Principles of modern lipid-lowering therapy. Lesson 9. Topic “Insulin therapy for type 2 diabetes”

Reasons for prescribing insulin for type 2 diabetes. Mechanism of action of insulin.

Types of insulin preparations and systems for administering insulin. Insulin therapy regimens. Injection technique training.

Complete the assignment for independent work

Task 3.13. Analyze a case from clinical practice. Patient K., 58 years old, is seen by a cardiologist at his place of residence with a diagnosis of coronary heart disease, exertional angina pectoris class II FC. Permanent form of atrial fibrillation. Stage 3 arterial hypertension, very high risk. Diffuse nodular goiter, euthyroidism. Gastric ulcer, remission. Due to the presence of a permanent form of atrial fibrillation in the patient, the cardiologist prescribed warfarin to prevent thromboembolic complications; under the control of the INR, a dose of warfarin of 7.5 mg per day was selected, against the background of which the INR was 2.37-2.5. No abdominal pain or hemorrhages were noted. 3 months after starting warfarin, the patient was transported by ambulance from a public place to the hospital due to developing pain in the precordial area (total duration of the attack 10 minutes), relieved after 4 times of using nitroglycerin spray (according to the patient). When receiving data for acute coronary syndrome, there was no clinical picture of hemorrhagic syndrome. The patient was examined by general practitioners from the emergency department and the therapist on duty. An examination (clinical blood test, general urine test, Wasserman reaction, ECG, biochemical blood test, INR) and treatment (isosorbide dinitrate, warfarin - 7.5 mg per day, metoprolol, enalapril, indapamide) were prescribed. However, the patient left the department on his own that same day after being examined by the therapist on duty, and therefore the prescribed examination and treatment was not started. A month later, the patient was taken by ambulance to the hospital with symptoms of hemorrhagic syndrome: subcutaneous hemorrhages, subconjunctival hemorrhage, hematuria, the INR was 9.8. According to the patient, over the last month he had been taking warfarin at the same dose (7.5 mg per day), but did not control the INR (he missed a scheduled INR measurement) because he was out of town and did not see a doctor. The patient also claims that he has been abusing alcohol for the past month.

In the hospital, warfarin was discontinued along with a transfusion of fresh frozen plasma and the use of vitamin K. The hemorrhagic syndrome was relieved, the INR decreased and upon discharge was 1.12. The patient was discharged in satisfactory condition with recommendations not to take warfarin due to the inability to control the INR.

1. What actions of the patient led to the development of hemorrhagic syndrome?

2. What are the reasons for the patient’s low compliance?

3. How can you avoid the development of hemorrhagic syndrome in a patient?

Clinical pharmacology. General issues of clinical pharmacology. Workshop: textbook. Sychev D.A., Dolzhenkova L.S., Prozorova V.K. and others / Ed. V.G. Kukesa. 2013. - 224 p.: ill.

Topic: Introduction

test 1. The science that studies drugs, their effect on a living organism with the aim of using them to treat diseases and prevent diseases is called:

1) pharmacology 3) pharmacodynamics

2) pharmacokinetics 4) pharmaceuticals

2. Pharmacotherapy, which is used to eliminate the cause of the disease or weaken its effect

1) pathogenetic 3) etiotropic

3. Pharmacology, which is aimed at eliminating or weakening the most dangerous symptoms is called:

1) pathogenetic 3) etiotropic

2) symptomatic 4) preventive

4. Pharmacology that is aimed at normalizing metabolic processes and physiological functions is called:

1) pathogenetic 3) etiotropic

2) symptomatic 4) preventive

5. The active principles of drugs include:

1) glycosides 3) microorganisms

2) plants 4) alkaloids

6. The science that studies the raw materials from which medicines are prepared is called:

1) therapy 3) pharmacognosy

2) pharmacotherapy 4) pharmacology

7. Who associated health with the balance of four fluids in the body: blood, mucus, black bile and yellow bile:

1) Galen 3) Hippocrates

2) Paracelsus 4) Avicenna

8. Who first introduced the concept of doses:

1) Galen 3) Hippocrates

2) Paracelsus 4) Avicenna

9. Products of natural and synthetic origin or a mixture of them, which are used for the diagnosis and treatment of animal diseases:

1) tablets 3) medicines

2) solutions 4) plants

10. Medicines that belong to list A in Latin are called:

2) Venena

11. The Heroica list includes the following medicines:

2) potent

3) drugs

12. What changes in the body of animals are caused by drugs:

1) anatomical

2) clinical

3) genetic

13. The Venena list includes the following medicinal substances:

1) poisons 3) anesthetics

2) analgesics 4) all drugs

14. Absorption of drugs into the blood through various routes of administration:

1) cumulation

2) absorption

3) generation

test No. 15. The removal of drugs from the body is called:

1) excretion

2) absorption

3) generation

16. What science studies prescription writing:

1) prescription

2) recipe

3) pharmacology

17. A scientist who considered disease as an imbalance in the body between atoms and the pores through which atoms move:

1) Galen 3) Shepkin

2) Ivan the Terrible 4) Asklepides

18. Science that studies the negative effects of drugs on the body:

1) pathology 3) toxicology

2) anatomy 4) epizootology

19. The direction of pharmacotherapy in which biostimulants, enzymes, hormones are used:

1) pharmacostimulation

2) therapy

3) physiotherapy

20. Inactivation, the biochemical mechanism of drug transformation in the body is called:

1) excretion

2) absorption

3) biotransformation

Topic: Pharmacy

Correct answers in the test are underlined

1. Where are explosive substances stored?

1) in the basement

2) in an aseptic block

3) in a separate closet

2. In what container are acids stored?

1) glass 3) cast iron

2) plastic 4) copper (bronze)

3. To obtain an acid solution:

1) acid is added to water

2) water is added to the acid

3) the order of mixing does not matter

4. In what container is silver nitrate stored?

1) glass with black paper

2) aluminum cans

3) galvanized cookware

5. Is it necessary to have sterile air in a chemical analytical room?

1) if possible yes

2) optional

3) required

6. What area should the sterilization room be?

1) 20 m 3) at least 8 m

2) at least 4 m 4) 30-40 m

7. The veterinary pharmacy has one room with an area of ​​at least:

1) 70 m 3) 20 m

2) 50 m 4) 10 m

8 test. In which room are all dosage forms checked?

1) chemical-analytical

2) aseptic

3) cubic

9. In which room are dosage forms made?

1) aseptic 3) still

2) material 4) assistant

10. In the pharmacy the air temperature is maintained:

1) 10 C 3) 23-25 ​​C

2) 18 C 4) up to 7 C

11. Poisonous substances in Latin:

2) Venena

12. Potent substances in Latin:

1) Heroica

13. Is it allowed to store potent and mildly potent substances in the same room:

1) No 2) yes 3) sometimes

14. Is it allowed to store toxic substances in a pharmacy in the absence of a special room?

1) sometimes 2) no 3) yes

15. Do all drugs need to be dispensed with a doctor's prescription?

1) yes 3) only list B

2) list A only 4) no

Topic: Recipe. Rules for writing prescriptions

Correct answers in the test are underlined

1. Written request from a doctor to a pharmacist:

1) note 3) recipe

2) statement 4) explanatory

2. Size of the form on which the prescription is written:

1) 100 x 200 3) 150 x 100

2) 105 x 150 4) 150 x 150

3. The largest number of prescriptions that can be written on one form:

1) 2 - 3 3) no more than 5

4. If the medications are from the Venena group, then on one form you can write out:

1) 2 - 3 3) no more than 5

2) 1 4) up to 10

5. If the recipe does not fit on one side of the form, then write below:

1) cito 3) perevertete

2)vertae 4)obrate

6. “Urgent” in Latin:

1) statim 3) perevertete

2) cito 4) obrate

7. “Very urgent” in Latin:

1) statim 3) cito

2) citisime 4) obrate

8 - test. "Immediately" in Latin:

1) statim 3) cito

2) citisime 4) obrate

9. How to correctly spell “antidote” in Latin:

1) anthidotum 3) antidotum

2) antidote 4) antidotes

10. “Repeat” in Latin:

1) repeticio 3) ripiticia

2) repetere 4) repere

11. Period for which poisons and narcotic drugs are prescribed:

1) 1 day 3) 3 days

2) 5 days 4) 2 months

12. The title in the recipe is:

1) inskripcio 3) desegnatio

2) prepositio 4) supscriptio

13. If the medicine in the prescription is prescribed in the same quantity, then write:

2) q.s. 4) et

14. Conjunction “And” in Latin:

3) q. s. 4) et

15. “Take as much as you need” in Latin:

1) ut. f. 3) ete

2) q. s. 4)et

16. “Mix” in Latin:

1) M. f. 3) N.f.

2) Ut. f. 4) Inf. f.

17. Excipients in a recipe are written in:

1) 1st place 3) 3rd place

2) 2nd place 4) last

18. What is written in the recipe after “quantum satis”:

1) Ut. f. 3) N. f.

2) M. f. 4) Inf. f.

19. “Let so many doses be given out” in Latin:

1) D. s. 3) D.t. d. No.

2) D. t. 4) D.Numero

recipe 20. “Water” in Latin:

1) aqua 3) aqua

2) aquae 4) aquaus

21. “Divide the doses so much” in Latin:

1) Div. In part. aeq No.

22. Two types of prescriptions:

1) divisional 3) deratization

2) dispensing 4) prescription

23. Drugs that are prescribed to improve taste or smell are called:

1) basis 3) corrigens

2) adiuvas 4) constituens

24. Instructions for the use of drugs are called:

1) subscription 3) inscription

2) signature 4) recipe

25. “Take” in Latin:

1) repete 3) recipeter

2) recipe 4) repeticio

26. What language is written in Subscripcio:

1) in native

2) in Latin

3) any

27. What language is written in the signature:

1) in native

2) in Latin

3) any

28. What language is written in Designatio materiarum:

1) in native

2) in Latin

3) any

29. What incompatibilities exist in the recipe:

1) biological 3) genetic

2) chemical 4) structural

30 - recipe. If the drugs specified in the prescription act in different directions after use, or one substance weakens the effect of another, then they speak of incompatibility:

1) pharmacological

2) chemical

3) physical

4) biological

Topic: Methods and types of action of pharmacological agents.

Correct answers in the test are underlined

1. The general effect of drugs after their absorption through different routes of administration is called:

1) local action 3) selective action

2) resorption 4) cumulation

2. Undesirable effects of drugs that stimulate the tumor process:

1) teratogenic 3) carcinogenic

2) mutagenic 4) embryotoxic

3. Undesirable effects manifested by defects in embryo development:

1) teratogenic 3) carcinogenic

2) mutagenic 4) emryotoxic

4. Acute addiction to drugs:

1) anaphylaxis 3) tachyphylaxis

2) paraphylaxis 4) metaphylaxis

5. The process of accumulation of medicinal substances in the body:

1) habit 3) paraphylaxis

2) addiction 4) cumulation

6. Simultaneous action of two or more medicinal substances in different directions:

1) antagonism 3) parargism

2) synergism 4) metargism

7. Simultaneous action in one direction

1) antagonism 3) parargism

2) synergism 4) metargism

8. The inhibitory effect of drugs on the central nervous system is characterized by:

1) paralysis 3) irritation

2) sleep 4) stimulation

9. Personal intolerance to drugs:

1) cumulation 3) idiosyncrasy

2) withdrawal 4) synergy

10. What types of antagonism exist:

1) physical 3) physiological

2) chemical 4) all of the above

pharmacology 11. Weakening of the effect with prolonged use of the drug:

1) habit 3) cumulation

2) addiction 4) idiosyncrasy

12. Action in which a medicinal substance acts directly on a cell or organ:

1) direct 3) local

2) indirect 4) resorptive

13. The effect of medicinal substances on individual organs and organ systems:

1) direct 3) selective

14. A group of drugs that have a predominantly local effect:

1) emollient 3) adsorbent

2) enveloping 4) all of the above

15. Is absorption of medicinal substances necessary:

1) yes

3) undesirable

16. What is observed with an overdose of stimulants

1) excitement

2) oppression

3) excitement then depression

17. What reaction of the body underlies allergies to medicinal substances:

1) sensitization

2) desensitization

3) idiosyncrasy

18. Which medicinal substances can directly cause allergies?

1) protein

2) not protein

3) synthetic

19. What drugs are used to treat drug allergies:

1) suprastin 3) prednisolone

2) diphenhydramine 4) all of the above

20. Addiction to substances such as opium, cocaine, alcohol, etc. causes cravings for them. What is the name of the state of the body (which has used these substances) when their use is abruptly stopped?

1) convulsions 3) withdrawal

2) coma 4) Budunism

Topic: Dose, dosing of medications

Correct answers in the test are underlined

No. 1. For oral administration of a certain drug, take a dose of 1. How much of the same drug is administered rectally, in relation to the IV dose.

1) 1/2 3) 1,5-2

2. IM dose in relation to the oral dose

1) 1/2 - 1/3 3) 1/4 - 1/5

2) 1/3 – 1/4 4) 1/5 – 1/6

3. SC dose in relation to oral dose

1) 1/2 - 1/3 3) 1/4 - 1/5

2) 1/3 – 1/4 4) 1/5 – 1/6

4. IV dose in relation to oral dose

2) 1/7 4) 1/4

5. Dose that will cause a therapeutic effect

1) preventive 3) therapeutic

2) lethal 4) maximum

6. Dose that will cause pathological changes in the body

1) lethal 3) toxic

2) curative 4) lethal

7. Therapeutic index formula

1) LD 50/TD 50

2) LD 100 / LD 50

3) LD 50 / LD 100

8. A dose higher than the therapeutic dose is used at the beginning of treatment to form the required concentration in the blood

1) Shock 3) primary

2) threatening 4) initial

9. Single dose in Latin:

1) d. edinidis 3) d. сoctes

2) d. prodosis 4) d. races

10. Daily dose in Latin:

1) d. prodie 3) d. sons

2) d. edinidis 4) d. sutes

11. Dose for a dog (weight 10 kg) in relation to the dose for a horse (weight 600 kg), taken as 1.

1) 0.2 – 0.25 3) 0.7 – 0.8

2) 0.5 4) 0.08 – 0.1

12. Dose for a pig (70 kg) in relation to the dose for a horse (600 kg)

1) 0.7 3) 0,16 – 0.2

2) 0.33 – 0.43 4) 0.02 – 0.03

13. Dose for sheep (60 kg) in relation to dose for horse (600 kg)

1) 0.2 – 0.25 3) 0.9 – 0.97

2) 0.17 – 0.18 4) same

14. Dose for a cat (2 kg) in relation to the dose for a horse (600 kg)

1) 0,02 – 1 3) 0.4 – 0.9

2) 0.02 – 0.05 4) 0.4 – 0.73

dose 15. Dose for chicken (2kg) in relation to dose for horse (600kg)

1) 0.02 – 0.05 3) 0.4 – 0.73

2) 0.2 – 1 4) 0.4 – 0.9

Topic: Drug poisoning.

Correct answers in the test are underlined

1. Types of poisoning according to the course of action:

1) acute 3) subacute

2) non-acute 4) chronic

2. If the contents of the intestines smell of garlic, then this is poisoning:

1) alcohol 3) copper

2) zinc phosphide 4) water

3. If the stomach and intestines have a grayish-black color, then this is poisoning:

1) lead 3) nitrates

2) copper 4) alcohol

4. If the color of the stomach is bluish-green, this is poisoning:

1) lead 3) copper

2) nitrates 4) alcohol

5. The antidote is otherwise called:

1) antibiotic 3) antidose

2) antidote 4) medicines

6. For formaldehyde poisoning use:

1) ammonium carbonate

2) formalin

3) calcium chloride

7. In case of poisoning with cholinomimetics, use:

1) atropine sulfate

2) prozerin

3) arecoline hydrochloride

8. For allergies to medications, use:

1) vitamins 3) diphenhydramine

2) enzymes 4) piperazine

9. In case of poisoning with narcotic substances use:

1) caffeine 3) paracetamol

2) analgin 4) diphenhydramine

10. In case of sugar (sugar beet) poisoning, use:

2) insulin

3) atropine sulfate

test - 11. Which drug is a good antidote for many poisonings:

1) paracetamol 3) analgin

2) unithiol 4) novocaine

12. In order to restore breathing, the following is administered subcutaneously:

1) analgin

2) cordiamine

3) ammonia

13. In order to restore the functioning of the cardiovascular system, the following is administered intramuscularly:

1) analgin

2) adrenaline

3) atropine

14. To avoid absorption of acid into the skin, the latter is washed with water and then with a solution:

1) 0.1% analgin

2) 0.1% formaldehyde

3) 0.1% sodium bicarbonate

15. Drugs that adsorb toxic substances include:

1) salt 3) flour

2) talc 4) white clay

Preview:

Subject: "Pharmaceutical drugs affecting the peripheral nervous system"

Test tasks

1.Adrenaline causes:

Choose one answer.

a.) Reduced oxygen consumption

B.) Hyperglycemia

C.) Inhibition of glycogenolysis

D.) Inhibition of lipolysis

2.Adrenaline is contraindicated for:

Choose one answer.

a.) Thyrotoxicosis

B.) Anaphylactic shock

C.) Heart block

D.) Hypoglycemic coma

3.Ganglion blocker:

Choose one answer.

a.) atropine;

B.) pipecuronium;

C.) pentamine;

D.) succinylcholine (dityline).

4.Ganglion blockers are used to treat:

Choose one answer.

a.) constipation.

B.) hypertensive crisis;

C.) glaucoma;

D.) urinary retention;

5. The action of the M-ChR agonist is blocked:

Choose one answer.

a.) Cytisine

B.) Tubocurarine

C.) Proserin

D.) Atropine

E.) Pilocarpine

6. To stop the action of competitive muscle relaxants, the following is used:

Choose one answer.

a.) atropine;

B.) dipyroxime.

C.) neostigmine (proserine);

7.Selective M-cholinomimetic (agonist of muscarinic cholinergic receptors):

Choose one answer.

a.) Prozerin

B.) Pilocarpine

C.) Cytisine

D.) Physostigmine

E.) Carbacholine

8.Adsorbing agents include:

Choose one answer.

a.) Starch mucus.

B.) Decoction of oak bark;

C.) Tannin;

D.) Activated carbon;

9. Irritants include everything except:

Choose one answer.

a.) Bismuth nitrate basic;

B.) Menthol.

C.) Purified turpentine oil (turpentine);

D.) Mustard paper;

10.M-anticholinergic:

Choose one answer.

a.) pentamine;

B.) pipecuronium;

C.) succinylcholine (dityline).

D.) atropine;

11.M-anticholinergics cause the development of mydriasis:

Choose one answer.

a.) increasing the tone of the radial muscle of the iris;

B.) reducing the tone of the orbicularis iris muscle;

C.) increasing the tone of the ciliary muscle.

12.M-anticholinergics are used to treat:

Choose one answer.

a.) arterial hypertension;

B.) glaucoma;

C.) gastric ulcer.

D.) myasthenia;

13.M-anticholinergics are contraindicated for:

Choose one answer.

a.) bronchial asthma;

B.) glaucoma;

C.) atrioventricular block;

D.) gastric ulcer.

14.M-cholinomimetics, unlike ChE inhibitors, do not affect cholinergic synaptic transmission:

Choose one answer.

a.) at the neuromuscular junction

B.) from postganglionic axons of autonomic nerves to the effector (smooth muscle, exocrine glands)

C.) in the central nervous system

15. Local anesthetics are used in combination with adrenaline, because:

Choose one answer.

a.) absorption of the anesthetic is accelerated and the local anesthetic effect is enhanced.

B.) absorption of the anesthetic slows down and the local anesthetic effect increases;

C.) absorption of the anesthetic slows down and the local anesthetic effect is weakened;

16. Metoprolol is indicated for the treatment of:

Choose one answer.

a.) Atrioventricular block

B.) Bronchial asthma

C.) Benign prostatic hyperplasia

D.) Arterial hypertension

17. The mechanism of action of astringents is due to:

Choose one answer.

a.) block of sodium channels;

B.) adsorption of chemical compounds;

C.) covering the mucous membranes with a film that prevents irritation of the sensory nerves. d.) coagulation of proteins and the formation of a film that protects the endings of sensory nerves from irritation;

18. The mechanism of action of local anesthetics is due to:

Choose one answer.

a.) Block of calcium channels and prolongation of the absolute refractory period;

B.) Block of potassium channels and inability to repolarize the membrane;

C.) Activation of chloride channels and hyperpolarization.

D.) Block of sodium channels and inability to depolarize the membrane;

19.Muscle relaxant:

Choose one answer.

a.) scopolamine.

B.) pipecuronium;

C.) atropine;

D.) pentamine;

20. Undesirable side effect of ganglion blockers:

Choose one answer.

a.) hypertensive crisis;

B.) increased intraocular pressure.

C.) orthostatic collapse;

D.) bronchospasm;

21.Non-selective beta blocker:

Choose one answer.

a.) Metoprolol

B.) Atenolol

C.) Prazosin

D.) Propranolol

22. Neostigmine (proserine) is used to treat myasthenia gravis because it improves cholinergic synaptic transmission:

Choose one answer.

a.) In the autonomic ganglion

B.) At the myoneural junction

C.) From postganglionic cholinergic fibers to cells of effector organs

23.Norepinephrine increases:

Choose one answer.

a.) Peripheral vascular resistance

B.) Motility of the gastrointestinal tract

C.) Bronchial tone

D.) Heart rate

24. Why is atropine (tertiary amine) superior to metacin (quaternary ammonium compound) in its action on the central nervous system:

Choose one answer.

a.) better distributed throughout the body (> Vd values);

B.) is better absorbed into the systemic circulation from the injection site (> bioavailability coefficient);

C.) is excreted (eliminated) from the body more slowly (> T1/2 values).

25. Why galantamine (tertiary amine) is superior to proserine (quaternary ammonium compound) in its action on the central nervous system:

Choose one answer.

a.) Slower eliminated from the body (> T1/2 values)

B.) Better absorbed into the systemic circulation from the injection site (> bioavailability coefficient)

C.) Better distributed throughout the body (>Vd values)

26.Prazosin causes:

Choose one answer.

a.) Decreased bronchial smooth muscle tone

B.) Reduced intraocular pressure

C.) Decrease and weakening of heart contractions

D.) Decrease in peripheral vascular resistance

27. Propranolol causes:

Choose one answer.

a.) Decreased gastrointestinal motility

B.) Contraction of the iris muscle (mydriasis)

C.) Decreased bronchial tone

D.) Decreased heart rate

28. Reflex bradycardia is caused by:

Choose one answer.

a.) Salbutamol

B.) Prazosin

C.) Metoprolol

D.) Carvedilol

E.) Norepinephrine

29. The contractile activity of the myometrium is reduced by:

Choose one answer.

a.) Salbutamol

B.) Carvedilol

C.) Propranolol

D.) Norepinephrine

E.) Metoprolol

30.Medication for the treatment of acute vascular insufficiency:

Choose one answer.

a.) Metoprolol

B.) Salbutamol

C.) Dobutamine

D.) Norepinephrine

E.) Propranolol

31. Only for superficial anesthesia is used:

Choose one answer.

a.) Benzocaine (anesthetic).

B.) Bupivacaine;

C.) Procaine (Novocaine);

D.) Lidocaine;

32. Cholinomimetics are contraindicated for:

Choose one answer.

a.) Myasthenia gravis

B.) Alzheimer's disease

C.) Bronchial asthma

D.) Xerostomia

E.) Glaucoma

Preview:

Subject: "Chemotherapeutic agents"

Test tasks

1. The mechanism of action of antigens on protein synthesis in microbial cells is based on their ability to inhibit:

Choose one answer.

a.) DNA polymerase

B.) RNA polymerase

C.) transpeptidation process

D.) the process of reading the m-RNA code

2. Select an antibiotic - an inhibitor of protein synthesis in a bacterial cell:

Choose one answer.

a.) benzylpenicillin

B.) carbenicillin

C.) ampicillin

D.) gentamicin

3.Select the definition of “chemotherapy”

Choose one answer.

a.) chemotherapy is the suppression of pathogens on the surface of the human body (skin, mucous membranes)

B) Chemotherapy is the suppression of pathogens in the environment (care items, instruments, patient secretions)

C.) chemotherapy is an effect on the cells of the macroorganism

D.) chemotherapy is the suppression of pathogens in the internal environments of the macroorganism

4.The principles of chemotherapy include the following:

Choose one answer.

a.) all answers are correct

B.) treatment with antibacterial drugs should be started as soon as possible after the onset of the disease

C.) the drug should be selected taking into account the sensitivity of the pathogen to the chemotherapeutic agent;

D.) the dose of chemotherapy should be prescribed taking into account the severity of the patient's disease

5. Nitrofuran derivatives include:

Choose one answer.

a.) Phthalylsulfathiazole (phthalazole)

B.) nalidixic acid

C.) furazolidone

D.) nitrohexoline

6.Which of the following antibiotics disrupt cell wall synthesis:

Choose one answer.

a.) chloramphenicol

B.) tetracyclines

C.) beta-lactam antibiotics

D.) polymyxins

7.Which drugs for the treatment of onychomycosis give the lowest relapse rate?

Choose one answer.

a.) terbinafine and itraconazole;

B.) amphotericin B and nystatin;

C.) griseofulvin and levorin;

D.) zinc undecylinate and iodine

8.What drugs are effective against respiratory syncytial viruses and influenza viruses?

Choose one answer.

a.) ribavirin, interferon;

B.) oseltamivir, remantadine

C.) azidothymidine, saquinavir;

D.) acyclovir, famciclovir;

9.Which of the statements correctly reflects one of the general principles of chemotherapy for an infectious disease.

Choose one answer.

B.) clinical improvement is grounds for discontinuation of therapy

C.) the effectiveness of treatment often does not depend on the duration of antibiotic therapy. d. 3) after clinical improvement, treatment should not be stopped and continued for another 48-72 hours if necessary.

10.Which antibiotic is effective for pseudomembranous colitis?

Choose one answer.

a.) dicloxacycline

B.) furazolidone

C.) vancomycin

D.) ampicillin

11.Which of the following antibiotics is a beta-lactam:

Choose one answer.

a.) streptomycin

B.) meropenem

C.) tetracycline

D.) polymyxin

12.Which of the chemotherapeutic drugs belongs to sulfonamides:

Choose one answer.

a.) mencomycin

B.) erythromycin

C.) streptomycin

D.) sulfadimine

13.Which macrolide has the lowest clearance?

Choose one answer.

a.) erythromycin

B.) azithromycin

C.) clarithromycin

D.) roxithromycin

14.Which oral medication is effective for fungal meningitis (eg, cryptococcal)?

Choose one answer.

a.) amphotericin B;

B.) fluconazole

C.) terbinafine;

D.) ketoconazole;

15.Which benzylpenicillin preparation is classified as biosynthetic:

Choose one answer.

a.) ampicillin

B.) benzylpenicillin-benzathine

C.) azlocillin

D.) carbenicillin

16.What drug is used to treat gastrointestinal candidiasis?

Choose one answer.

a.) clotrimazole

B.) griseofulvin;

C.) nitrofungin;

D.) nystatin;

17.What drug is used to treat systemic mycoses?

Choose one answer.

a.) nystatin;

B.) clotrimazole

C.) amphotericin B;

D.) griseofulvin;

18.Which drug is effective against influenza viruses type A and B?

Choose one answer.

a.) remantadine

B.) acyclovir;

C.) azidothymidine;

D.) oseltamivir;

19. The mechanism of action of sulfonamides is associated with:

Choose one answer.

a) inhibition of COX

B.) inhibition of dihydrofolate reductase;

C.) competitive antagonism with PABA and inhibition of dihydropteroate synthetase

D.) competitive antagonism with GABA

20. The most common complication of beta-lactam antibiotics is

Choose one answer.

a.) arrhythmias

B.) inhibition of hematopoiesis

C. 1) allergic reactions

D.) hearing loss

21.Polymyxins are considered as 3rd line (“deep reserve”) drugs because:

Choose one answer.

a.) have low efficiency

B.) widespread resistance to them

C.) due to low antimicrobial activity

D.) due to high organotoxicity

22. When using resorptive sulfonamides, the following side effects are possible:

Choose one answer.

a.) agranulocytosis

B.) all of the above

C.) crystalluria

D.) hemolytic anemia, methemoglobinemia

23. Antiviral drugs (AVS) are most effective when treatment is started early, because:

Choose one answer.

a.) PVA exhibit a vistatic effect;

B.) PVA exhibit a viricidal effect;

C.) PVA does not exhibit organotoxicity

D.) PVA is organotoxic;

24. Specify antiretroviral drugs (for the treatment of HIV infection):

Choose one answer.

a.) arbidol, oseltamivir;

B.) azidothymidine, saquinavir;

C.) acyclovir, famciclovir;

D.) interferon, ganciclovir

25. Specify the mechanism of action of fluoroquinolones:

Choose one answer.

a.) increasing the permeability of the CPM

B.) Inhibition of bacterial wall synthesis

C.) inhibition of PDEase

D.) inhibition of DNA gyrase

26. Specify a drug related to oxazolidinones:

Choose one answer.

a.) linezolid

B.) moxifloxacin

C.) co-trimoxazole

D.) lincomycin

27. Specify an antiherpetic agent:

Choose one answer.

a.) azidothymidine;

B.) acyclovir;

C.) arbidol;

D.) saquinavir

28.What is characteristic of doxycycline?

Choose one answer.

a.) poorly absorbed from the gastrointestinal tract

B.) bioavailability decreases when taken with food

C.) T1/2 16-24 hours

D.) the main route of elimination is through the MVP

Preview:

Subject : "General Pharmacology"

Test tasks

1 . Substances with affinity and internal activity are called:

Choose one answer.

a.) antagonists

B.) agonists

2 . The action of substances that develops after its entry into the systemic circulation is called:

Choose one answer.

a.) resorptive

B.) local

C.) by-product

D.) reflex

3 . What is the action of a substance called if it interacts only with functionally unambiguous receptors of a certain localization and does not affect other receptors?

Choose one answer.

a.) reflex

B.) reversible

C.) irreversible

D.) selective

4 . What is the accumulation of medicinal substances in the body during repeated administration called?

Choose one answer.

a.) tachyphylaxis

B.) material cumulation

C.) idiosyncrasy

D.) sensitization

5 . What is the decrease in the effectiveness of a substance called when it is repeated?

Choose one answer.

a.) tolerance (addiction)

B.) cumulation

C.) idiosyncrasy

D.) addiction

6.What is the name of the phenomenon when drug withdrawal causes mental and somatic disorders associated with dysfunction of many body systems, even death?

Choose one answer.

a.) withdrawal syndrome

B.) withdrawal

C.) sensitization

D.) idiosyncrasy

7.Which of the processes occurs during the biotransformation phase, which is called conjugation?

Choose one answer.

a.) hydrolysis

B.) recovery

C.) acidification

D.) acetylation

8.Which answer best matches the term “receptor”?

Choose one answer.

a.) active groups of substrate macromolecules with which the drug interacts

B.) drug-activated transport systems

C.) drug-activated redox enzymes

D.) ion channels of biological membranes, the permeability of which is changed by the drug substance

9.Which pharmacokinetic parameter is designated as “T1/2”:

Choose one answer.

a.) elimination rate constant

B.) half-life (half-life, half-elimination) of substances

C.) absorption from the injection site of 50% of the substance

D.) total ground clearance

10.Metabolic biotransformation is:

Choose one answer.

a.) interaction with glucuronic acid

B.) transformation of a substance due to oxidation, reduction, hydrolysis

C.) binding to blood plasma albumin

D.) methylation and acetylation of substances

11.The volume of distribution of drugs reflects:

Choose one answer.

a.) the ratio of single and daily doses of a medicinal substance

B.) hypothetical volume of fluid into which the drug is distributed

C.) the calculated amount of drug that reaches the systemic circulation

D.) dose-body weight ratio

12.The volume of distribution is low if:

Choose one answer.

a.) the substance is found in plasma, interstitial and intracellular fluid and accumulates in tissues

B.) the substance is found in plasma and interstitial fluid

C.) the substance is found in plasma, interstitial and intracellular fluid

D.) the substance accumulates in the blood plasma

13.Note the main mechanism of absorption of drugs:

Choose one answer.

a.) pinocytosis

B.) passive diffusion

C.) active transport

D.) filtering

14.Pharmacokinetics includes:

Choose one answer.

a.) biotransformation of medicinal substances in the body

B.) Effect of drugs on the genetic apparatus

C.) complications of drug therapy

D.) The effect of drugs on metabolism in the body

15.What does the concept of pharmacodynamics include?

Choose one answer.

a.) metabolism of drugs in the body

B.) storage conditions for medicines

C.) biological effects of drugs

D.) method of drug administration

16.What is included in the concept of “Biotransformation”:

Choose one answer.

a.) binding of substances to blood plasma proteins

B.) accumulation of substances in adipose tissue

C.) a complex of physicochemical and biochemical transformations of a medicinal substance aimed at removing it from the body

D.) accumulation of the drug in muscle tissue

17.What is the internal activity of a substance called?

Choose one answer.

a.) the ability of a substance, when interacting with a receptor, to recognize it

B.) the ability of a substance to interact with transport systems

C.) the ability of a substance, when interacting with a receptor, to stimulate it and cause a biological effect

D.) the ability of a substance to interact with plasma proteins

18.What does the term “affinity” mean?

Choose one answer.

a.) the affinity of the substance for the transport systems of the body

B.) affinity of the substance for blood plasma albumin

C.) affinity of drugs for microsomal liver enzymes

D.) the affinity of a substance for a receptor, leading to the formation of a “substance-receptor” complex with it

19.What does the term “bioavailability” mean:

Choose one answer.

a.) degree of binding of substances to plasma proteins

B.) the amount of substance in the urine relative to the initial dose of the drug

C.) ability to pass through the blood-brain barrier

D.) the amount of unchanged substance that reached the blood plasma, relative to the initial dose of the drug

20.What corresponds to the concept of “active transport”:

Choose one answer.

a.) invagination of the cell membrane with the formation of a vacuole

B.) transport against a concentration gradient with energy consumption

C.) transport along a concentration gradient without energy consumption

D. 1) facilitated diffusion

Preview:

Subject : “Immunotropic agents”

Test tasks

1. H1-histamine receptor blockers are used for all of the following indications EXCEPT:

Choose one answer.

a.) urticaria;

B.) bronchial asthma

C.) drug allergies;

D.) seasonal rhinitis;

2.What types of pharmacodynamic effects of glucocorticoids are used in medicine?

Choose one answer.

a.) all are correct except 1

B.) hyperglycemic, suppression of the growth zones of the epiphyses;

C.) anti-shock, detoxifying (induction of liver enzymes);

D.) all of the above;

E.) immunosuppressive, antiallergic, anti-inflammatory;

3.Which drugs are most effective as anti-inflammatory drugs?

Choose one answer.

a.) broad-spectrum antibiotics

B.) NSAIDs;

C.) SPVA;

D.) mast cell membrane stabilizers;

4.What are the acceptable criteria for the effectiveness of insulin therapy?

Choose one answer.

a.) euglycemia, euglucosuria;

B.) euglycemia, aglucosuria;

C.) aglycemia, aglucosuria

D.) normoglycemia, euglucosuria;

5. What are the local undesirable effects with the systematic use of GCS-containing ointments and creams?

Choose one answer.

a.) swelling, hyperemia, pain;

B.) osteoporosis, hirsutism, dysmenorrhea

C.) hypertrophy, hyperpigmentation, candidiasis;

D.) increased risk of local infections, atrophy, depigmentation;

6.What are the most dangerous undesirable effects of systemic corticosteroids during long-term use?

Choose one answer.

a.) withdrawal syndrome (adrenal insufficiency);

B.) all of the above;

C.) Itsenko-Cushing syndrome (“Cushingoid”);

D.) 1 and 2 are correct.

E.) immunodeficiency state;

7.What are the indications for the use of gestagen drugs?

Choose one answer.

a.) hormone replacement therapy after ovariohysterectomy;

B.) breast cancer, prostate cancer;

C.) dysfunctional uterine bleeding, recurrent miscarriage, endometriosis, contraception;

D.) contraception in women with high hyperdyslipidemia, diabetes mellitus, and a history of cholestasis

8.What complication is the most common when treated with insulin drugs?

Choose one answer.

a.) lipodystrophy;

B.) hypokalemia;

C.) insulin resistance

D.) hypoglycemia;

9.Which GCS preparation has low bioavailability when used topically (on the skin)?

Choose one answer.

a.) budesonide;

B.) fluocinolone acetonide (sinaphlan);

C.) prednisolone hemisuccinate

D.) beclamethasone propionate;

10.Which drug of inhaled corticosteroids provides the least risk of systemic undesirable effects?

Choose one answer.

a.) prednisolone hemisuccinate

B.) beclamethasone propionate;

C.) budesonide;

D.) fluocinolone acetonide (sinaphlan);

11.Which drug is an insulin sensitizer?

Choose one answer.

a.) metformin;

B.) humulin

C.) pioglitazone;

D.) acarbose;

E.) glibenclamide;

12.Which drug is used for seasonal allergic reactions (hay fever) only as a means of prevention?

Choose one answer.

a.) clemastine;

B.) hydrocortisone;

C.) sodium cromoglycate;

D.) all of the above

13. Second generation H1-histamine blockers differ from first generation drugs

Choose one answer.

a.) pronounced sedative effect;

B.) antiemetic effect

C.) significant M-anticholinergic effect;

D.) greater selectivity of action;

14. Oxytocin is characterized by all properties EXCEPT

Choose one answer.

a.) the sensitivity of the uterus is constantly high

B.) effective in small doses as a rhodostimulant;

C.) effective in large doses as a uterotonic;

D.) the sensitivity of the uterus to it increases during childbirth;

15. Indications for the use of immunosuppressive cytostatics include everything EXCEPT:

Choose one answer.

a.) autoimmune diseases;

B.) prevention of ROT

C.) severe anaphylactic reactions;

16. For hypothyroidism, it is used as a means of replacement therapy

Choose one answer.

a.) protirelin;

B.) potassium iodide;

C.) thyrotropin

D.) levothyroxine;

17. The use of cytostatic immunosuppressants (methotrexate, fluorouracil, cyclophosphamide) is often complicated

Choose one answer.

a.) leukopenia and infectious syndrome;

B.) allergies and photodermatitis;

C.) bleeding and anemia;

D.) drowsiness and lethargy

18. Thiamazole (Mercazolil) as a primary (long-term) treatment is indicated for…

Choose one answer.

a.) thyroid cancer;

B.) myxedema

C.) nodular toxic goiter;

D.) diffuse toxic goiter;

19.Indicate the correct sequence of use of drugs for anaphylactic shock:

Choose one answer.

a.) prednisolone – clemastine – aminophylline – epinephrine;

B.) clemastine (tavegil) - epinephrine (adrenaline) – prednisolone - aminophylline (aminophylline)

c.) epinephrine – prednisolone – clemastine – aminophylline

20. How to stop diabetic coma?

Choose one answer.

a.) IV 40-80 ml of 40% glucose solution;

B.) IV 1 ml of 0.1% adrenaline solution

C.) intravenous 20 units insulin-zinc suspension;

D.) intravenous short-acting insulin 0.1 units/hour;

21.What is NOT an absolute contraindication to the use of estrogen drugs?

Choose one answer.

a.) uterine bleeding of unknown nature;

B.) liver disease, history of jaundice;

E.) thrombophilia;

Preview:

Subject : “Medicines affecting

on the function of executive bodies"

Test tasks

1.Medication for emergency treatment of hypertensive crisis (when signs of target organ damage appear or increase):

Choose one answer.

a.) methyldopa;

B.) captopril;

C.) sodium nitroprusside

D.) metoprolol;

2.AAS for the treatment of supraventricular and ventricular tachyarrhythmias:

Choose one answer.

a.) Verapamil

B.) Lidocaine

C.) Procainamide (Novocainamide)

D.) Phenytoin (Difenin)

3.AAS with minimal potential for proarrhythmogenic action:

Choose one answer.

a.) Propranolol (Anaprilin)

B.) Amiodarone

C.) Propafenone

D.) Lidocaine

4.AAS used to treat coronary artery disease:

Choose one answer.

a.) Lidocaine

B.) Verapamil

C.) Quinidine

D.) Propafenone

5.AAS characterized by the longest half-life:

Choose one answer.

a.) Quinidine

B.) Adenosine

C.) Lidocaine

D.) Amiodarone

6. Antihypertensive neurotropic agent of peripheral action:

Choose one answer.

a.) captopril;

B.) metoprolol;

C.) nifedipine

D.) clonidine;

7. Antihypertensive drug from the group of calcium channel blockers:

Choose one answer.

a.) nifedipine

B.) metoprolol;

C.) captopril;

D.) losartan;

8. Antihypertensive drug from the group of myotropic vasodilators:

Choose one answer.

a.) captopril;

B.) diltiazem;

C.) dichlorothiazide;

D.) metoprolol

9. Centrally acting antihypertensive agent:

Choose one answer.

a.) clonidine;

B.) pentamine

C.) sodium nitroprusside;

D.) captopril;

10. Antihypertensive agent that blocks alpha and beta adrenergic receptors: .

Choose one answer.

a.) carvedilol;

B.) metoprolol

C.) atenolol;

D.) propranolol;

11. Antihypertensive drug for which there is a high risk of developing the effect of the first dose (severe hypotension in the orthostatic position):

Choose one answer.

a.) metoprolol;

B.) hydrochlorothiazide;

C.) captopril;

D.) prazosin

12. Antihypertensive drug contraindicated in bilateral renal artery stenosis:

Choose one answer.

a.) metoprolol;

B.) verapamil;

C.) captopril;

D.) nifedipine

13. Antihypertensive agent that reduces the formation of angiotensin II:

Choose one answer.

a.) verapamil;

B.) losartan;

C.) captopril;

D.) prazosin

14. Antihypertensive agent that reduces the level of renin in the blood:

Choose one answer.

a.) prazosin;

B.) verapamil;

C.) propranolol

D.) pentamine;

15. Has an antifibrinolytic effect:

Choose one answer.

a.) phytomenadione

B.) calcium chloride

C.) heparin

D.) aminocaproic acid

16. Beta-blockers are used for ischemic heart disease based on:

Choose one answer.

a.) reducing oxygen demand by reducing heart rate and myocardial contractility; b.) reducing myocardial oxygen demand by reducing preload;

C.) increased extraction of O2 from the blood

D.) improvement of coronary blood flow;

17. A substance that activates fibrinolysis is:

Choose one answer.

a.) Warfarin

B.) clopidogrel

C.) hirudin

D.) streptokinase

18. All cardiotonic drugs increase:

Choose one answer.

a.) atrioventricular conduction;

B.) myocardial contractility;

C.) automaticity of the sinoatrial node

D.) oxygen demand of the myocardium;

19. The main element of the antihypertensive action of alpha-blockers:

Choose one answer.

a.) Venous vasodilation;

B.) Negative chrono- and inotropic effects

C.) Arteriolar vasodilation;

20. The main element of the antihypertensive action of beta-blockers:

Choose one answer.

a.) Negative chrono- and inotropic effects

B.) Arteriolar vasodilation;

C.) Venous vasodilation;

D.) Blockade of angiotensin II receptors;

21. The following is NOT used for the treatment of arterial hypertension:

Choose one answer.

a.) furosemide

B.) spironolactone;

C.) hydrochlorothiazide;

D.) mannitol;

22. For the systematic treatment of arterial hypertension the following is NOT used:

Choose one answer.

a.) metoprolol;

B.) losartan;

C.) phentolamine

D.) nifedipine;

23. Bronchodilators from the group of sympathomimetics include:

Choose one answer.

a.) isadrin

B.) ephedrine

C.) salbutamol

24. Non-glycoside cardiotonics include all drugs EXCEPT:

Choose one answer.

a.) milrinone

B.) strophanthin (ouabaina);

C.) dopamine;

D.) dobutamine;

25.Indirect anticoagulant drugs include:

Choose one answer.

a.) hirudin

B.) Sodium hydrogen citrate

C.) fraxiparine

D.) warfarin

26. Cardiac glycosides (CG) include all of the following drugs, EXCEPT:

Choose one answer.

a.) digoxin;

B.) dobutamine;

C.) digitoxin;

D.) strophanthin

27.Which combination of diuretics is rational?

Choose one answer.

a.) Furosemide + mannitol

B.) Mannitol + urea

C.) Dichlorothiazide + triamterene

D.) Furosemide + ethacrynic acid

28.What nitroglycerin preparations do patients with angina use to relieve attacks?

Choose one answer.

a.) nitroglycerin in ointment

B.) nitroglycerin in sublingual tablets;

C.) nitroglycerin in microdrags (sustak);

D.) nitroglycerin in solution for intravenous administration;

29.What signs of SG overdose are life-threatening?

Choose one answer.

a.) fatigue, muscle weakness

B.) dyspeptic disorders;

C.) visual disturbances;

D.) ventricular tachyarrhythmias;

30.What drugs are used to treat ischemic heart disease?

Choose one answer.

a.) all of the above

B.) antiatherosclerotic agents;

C.) antithrombotic agents;

D.) cardioprotective agents;

31.What remedy can relieve intestinal spasms (colic)?

Choose one answer.

a.) metamizole (analgin);

B.) metoclopramide;

C.) drotaverine (no-spa).

D.) morphine;

E.) magnesium sulfate;

32.Which antacid can cause alkalosis when used systematically?

Choose one answer.

a.) magnesium trisilicate;

B.) aluminum hydroxide;

C.) calcium gluconate;

D.) sodium bicarbonate

E.) magnesium oxide;

33.What antiemetic is used for reflux and gastric paresis?

Choose one answer.

a.) ondansetron (zofran);

B.) chlorpromazine (aminazine);

C.) metoclopramide;

D.) diphenhydramine (diphenhydramine);

E.) perphenazine hydrochloride (etaperazine)

34.Which diuretic can cause hearing loss?

Choose one answer.

a.) spironolactone

B.) Dichlorothiazide

C.) mannitol

D.) furosemide

35.What drug is used to enhance the contractile activity of the myometrium during childbirth?

Choose one answer.

a.) ergometrine maleate

B.) Atropine sulfate

C.) oxytocin

D.) papaverine

36.What drug is used to stop uterine bleeding?:

Choose one answer.

a.) ergometrine maleate

B.) Atropine sulfate

C.) fenoterol

D.) prostaglandin F-2a

37.Which drug is a direct-acting anticoagulant?

Choose one answer.

a.) fibrinolysin

B.) phytomenadione

C.) Heparin

D.) Warfarin

38.Which drug is used only to prevent vomiting caused by motion sickness (motion sickness)?

Choose one answer.

a.) metoclopramide (cerucal);

B.) perphenazine hydrochloride (etaperazine)

C.) diprazine (pipolfen);

D.) "Aeron";

E.) ondansetron (Zofran);

39.Which drug reduces myometrial contractility?

Choose one answer.

a.) fenoterol

B.) pituitrin

C.) prostaglandin F-2a

D.) Papaverine

40. The mechanism of the bronchodilator action of atropine is associated with:

Choose one answer.

a.) direct myotropic effect on bronchial smooth muscles

B.) blockade of m-cholinergic receptors of bronchial smooth muscles

C.) stimulation of B2-adrenergic receptors

41. Mechanism of action of loop diuretics (furosemide, etc.):

Choose one answer.

a.) Increase the osmotic pressure of the fluid in the lumen of the tubules

B.) Reduce the reabsorption of sodium, chloride and potassium in the thick part of the ascending limb of the loop of Henley

c.) Increase glomerular filtration

D.) Block carbanhydrase

42. Mechanism of action of thiazide diuretics?

Choose one answer.

a.) Increase the osmotic pressure of fluid in the nephron tubules

B.) Increase glomerular filtration rate

C.) Block aldosterone receptors

d.) Reduce the reabsorption of sodium and chlorine in the distal tubules

43. The mechanism of the expectorant action of thermopsis preparations is due to:

Choose one answer.

a.) irritation of stomach receptors and reflex enhancement of the secretion of bronchial glands

b.) direct stimulation of the secretion of the bronchial glands

c.) liquefaction of sputum during depolymerization of proteins

44. The most suitable indication for prescribing SG is:

Choose one answer.

a.) unstable angina;

b.) CHF with severe bradycardia;

c.) CHF with multiple ventricular extrasystoles

d.) CHF with atrial fibrillation;

45. A common property of all AAS (except cardiac glycosides) used to treat tachyarrhythmias:

Choose one answer.

a.) Slowing down rapid depolarization

b.) Slowing down repolarization

c.) Acceleration of repolarization

d.) Reduced automaticity

46.The main properties of heparin are:

Choose one answer.

a.) Cumulates

b.) effective when taken orally

c.) Action develops after 18-24 hours

d.) Delays blood clotting “in vivo” and “in vitro”

47. Features of the action of lidocaine:

Choose one answer.

a.) Slows down rapid depolarization

b.) Accelerates repolarization

c.) Slows conduction

d.) Increases blood pressure

48.Note the antiplatelet agent - COX inhibitor:

Choose one answer.

a.) warfarin

b.) phytomenadione

c.) acetylsalicylic acid

d.) Sodium hydrogen citrate

49.Note the direct acting coagulant:

Choose one answer.

a.) phytomenadione

b.) thrombin

c.) aprotinin

d.) Heparin

50. Note the side effects of euphilin:

Choose one answer.

a.) respiratory depression

b.) increased myocardial oxygen demand

c.) increased blood pressure

51. Mark the drug related to hydrouretics:

Choose one answer.

a. 2) indapamide

b. 3) mannitol

c. 1) Dichlorothiazide

d. 4) furosemide

52. Mark the drug related to saluretics:

Choose one answer.

a.) Urea

b.) Mannitol

c.) Demeclocycline

d.) Furosemide

53. Mark the drug that enhances the outflow of bile (cholekinetic):

Choose one answer.

a.) “Holenzyme”;

b.) dehydrocholic acid;

c.) drotaverine (no-spa);

d.) magnesium sulfate;

e.) atropine;

f.) aminophylline (aminophylline)

54. Note the cholesecretic drug of plant origin:

Choose one answer.

a.) Magnesium sulfate;

b.) osalmide (oxaphenamide);

c.) “Allohol”;

d.) "Holenzym"

e.) drotaverine (no-spa);

55. Mark a laxative for emergency bowel cleansing (preparation for therapeutic or diagnostic procedures):

Choose one answer.

a.) lactulose;

b.) magnesium sulfate;

c.) isafenin;

d.) glycerin suppositories;

e.) phenolphthalein

56. List the means of replacement therapy for chronic pancreatitis:

Choose one answer.

a.) kontrikal;

b.) pentagastrin

c.) misoprostol;

d.) atropine;

e.) pancreatin;

57. List a drug that reduces the secretion of hydrochloric acid in the stomach:

Choose one answer.

a.) omeprazole;

b.) sodium bicarbonate;

c.) aluminum hydroxide

d.) histamine;

e.) pentagastrin

58.Why can coronary lytic drugs (for example, dipyridamole) cause the “steal phenomenon” of the myocardium?

Choose one answer.

a.) tone the coronary vessels;

b.) increase myocardial contractility

c.) redistribute blood flow to healthy vessels to the detriment of the ischemic area of ​​the myocardium;

d.) dilate the vessels of the systemic circulation;

59. A drug that reduces bronchial reactivity from the GC group is used for bronchial asthma:

Choose one answer.

a.) beclamethasone dipropionate

b.) cromolyn sodium

c.) ipratropium bromide

60. The drug of choice for relieving bronchospasm is:

Choose one answer.

a.) isadrin

b.) salbutamol

c.) atropine

61.For what diseases are diuretics used for routine therapy:

Choose one answer.

a.) acute poisoning

b.) Cerebral edema

c.) hypertension

d.) Pulmonary edema

62. For pulmonary edema, the following is used to reduce pressure in the pulmonary circulation:

Choose one answer.

a.) ganglion blockers

b.) oxygen inhalation

c.) respiratory stimulants

63. For pulmonary edema, inhalation of an ethyl alcohol solution is used to:

Choose one answer.

a.) anti-foaming action

b.) narcotic effect

c.) dehydrating effect

64. The antiarrhythmic effect of cardiac glycosides is due to:

Choose one answer.

a.) Decreased strength of heart contractions

b.) Slowing of conduction

c.) Reduced automaticity

d.) Decreased excitability

65.An antitussive that suppresses the cough reflex and blocks the excitability of sensitive endings in the respiratory tract is:

Choose one answer.

a.) tusuprex

b.) codeine

c.) libexin

66. The following has a mixed stimulating effect on the respiratory center:

Choose one answer.

a.) caffeine

b.) niketamide (cordiamin)

c.) cititon

67. Specific undesirable side effect of angiotensin-converting enzyme inhibitors:

Choose one answer.

a.) dry cough

b.) agranulocytosis;

c.) rhinorrhea;

d.) anorexia;

68.Medicine for the treatment of bradyarrhythmias

Choose one answer.

a.) Verapamil

c.) Lidocaine

d.) Atropine

69.A remedy for the treatment of ventricular tachyarrhythmias only

Choose one answer.

a.) Propafenone

b.) Procainamide (Novocainamide)

c.) Lidocaine

d.) Verapamil

70.A remedy for the treatment of supraventricular tachyarrhythmias only

Choose one answer.

a.) Lidocaine

b.) Procainamide (Novocainamide)

c.) Verapamil

d.) Propafenone

71.Indicate the most common undesirable effect of nitrates:

Choose one answer.

a.) methemoglobin formation;

b.) headache;

c.) decreased tone of the gallbladder

d.) suppression of platelet aggregation;

72.What is the indication for metoclopramide?

Choose one answer.

a.) diarrhea;

b.) low acidity;

c.) increased acidity;

d.) kinetosis (sea, air sickness);

e.) nausea, vomiting.

Preview:

Subject:"Medicines affecting

on the central nervous system"

Test tasks

1.What is true about ASC?

Choose one answer.

a.) not used for arthritis;

b.) least ulcerogenic;

c.) do not use in children under 14 years of age with fever;

d.) as an antiplatelet agent used in larger than analgesic doses

2.What is NOT a contraindication for the use of opioid analgesics?

Choose one answer.

a.)respiratory depression;

b.) myocardial infarction;

c.) traumatic brain injury

d.) acute abdominal pain of unknown origin;

3.What is used to restore breathing in case of an overdose of heroin (morphine)?

Choose one answer.

a.)naloxone;

b.) oxygen;

c.)tramadol;

d.)naltrexone

4.What is characteristic of the antipyretic effect of non-narcotic analgesics?

Choose one answer.

a.) NAs cause hypothermia by suppressing heat production;

b.) appointment is mandatory for low-grade fever;

c.) NAs reduce fever by increasing heat transfer

d.) it is inherent in all NAs in doses significantly higher than analgesic ones;

5.What is characteristic of pyrazolone derivatives (metamizole (analgin), phenylbutazone (butadione))?

Choose one answer.

a.) used as antiplatelet agents for ischemic heart disease

b.) used for long-term therapy of arthritis;

c.) hematotoxic;

d.) no anti-inflammatory effect;

6.What is characteristic of the anti-inflammatory effect of NSAIDs?

Choose one answer.

a.) improve the quality of life of patients, temporarily reducing the symptoms of arthritis;

b.) cure arthritis with a full course of treatment;

c.) inhibit all phases of inflammation;

d.) anti-inflammatory effect is due to inhibition of leukotriene synthesis7.What is the cause of death in an overdose of opioid analgesics?

Choose one answer.

a.) bronchospasm;

b.) pulmonary edema;

c.) cessation of breathing;

d.) cardiac arrest

8. Extrapyramidal movement disorders - a typical undesirable side effect:

Choose one answer.

a.) Clozapine

b.) Haloperidol

c.) Olanzapine

d.) Risperidone

9.What drug is used for status epilepticus?

Choose one answer.

a.) diphenhydramine (diphenhydramine);

b.)diazepam;

c.)ethosuximide

10.What effect of opioid analgesics can be dangerous even with a single use against the background of severe pain

Choose one answer.

a.) spasms in the gastrointestinal tract;

b.)euphoria;

c.)constipation

d.)respiratory depression;

11.What effect of opioid analgesics limits their widespread use?

Choose one answer.

a.) analgesic;

b.)sedative;

c. 3) euphoric;

d. 4) spasmogenic

12.Caffeine:

Choose one answer.

a.) Tones the respiratory and vasomotor centers

b.) Constricts coronary vessels

c.) Dilates cerebral vessels

13. Moclobemide, compared to imipramine, has a stronger:

Choose one answer.

a.) Psychostimulating effect

b.) Psychosedative effect

c.) Alpha adrenergic blocking effect

d.) M-anticholinergic effect

14. Nimesulide and celecoxib - selective COX-2 inhibitors - differ from non-selective ones (ASA, diclofenac, etc.):

Choose one answer.

a.) greater efficiency;

b.) lower frequency of gastropathy;

c.) less allergenic;

d.) significantly lower severity of all “PG-dependent” side effects

15. Main indication for the use of opioid (narcotic) analgesics

Choose one answer.

a.)traumatic and visceral pain of high intensity

b.) traumatic and visceral pain of moderate intensity;

c.) neuralgia;

d.) osteoalgia;

16.Note the undesirable effects common to NA/NSAIDs (“COX- and PG-dependent”):

Choose one answer.

a.) addiction, drug dependence;

b.) lethargy, respiratory depression;

c.) allergic reactions, leukopenia

d.) gastropathy, bleeding;

17.Check the correct statement about ketorolac:

Choose one answer.

a.) effective only for moderate pain;

b.) used for long-term therapy of arthritis;

c.) used for no longer than 5-7 days due to nephrotoxicity

d.) hepatotoxic;

18. For parkinsonism the following is used:

Choose one answer.

a.)phenytoin (diphenin);

b.)carbamazepine;

c.)levodopa

19. Antiemetic effect has:

Choose one answer.

a.) Chlorpromazine

b.) Buspirone

c.) Zopiclone

d.) Diazepam

20. Anticonvulsant effect has:

Choose one answer.

a.) Haloperidol

b.) Diazepam

c.) Buspirone

d.) Chlorpromazine

21. Psychostimulant from the methylxanthine group:

Choose one answer.

a.) Amphetamine

b.) Caffeine

c.) Moclobemide

d.) Piracetam

e.) Imipramine

22. Specific treatment for acute poisoning with benzodiazepine derivatives:

Choose one answer.

a.) Zopiclone

b.) Flumazenil

c.) Caffeine

d.) Piracetam

e.) Phenazepam

23. Tranquilizer with the longest half-life (T1/2 >48 hours):

Choose one answer.

a.) Diazepam

b.) Oxazepam

c.) Lorazepam

d.) Medazepam

e.) Midazolam

24. Tricyclic antidepressant:

Choose one answer.

a.) Caffeine

b.) Amitriptyline

c.) Fluoxetine

d.) Piracetam

25. Indicate the features of the analgesic effect of NA:

Choose one answer.

a.) effective for moderate arthralgia, myalgia, cephalgia;

b.) eliminate pain of any intensity;

c.) more effective than narcotic analgesics for severe traumatic and visceral pain;

d.) with long-term use, tolerance develops

26.Fluoxetine compared with amitriptyline::

Choose one answer.

a.) Less toxic

b.) Has a stronger M-anticholinergic effect

c.) Characterized by greater clinical effectiveness

d.) Has a stronger sedative effect

27. How do hypnotic derivatives of barbituric acid differ from benzodiazepine derivatives?

Choose one answer.

a.) pronounced central muscle relaxant effect

b.) more disturbance of sleep structure;

c.) weak induction of microsomal liver enzymes;

28. How do partial agonists and agonists-antagonists of opioid receptors (pentazocine, buprenorphine) differ from full agonists (morphine)?

Choose one answer.

a.) stronger spasmogenic effect;

b.) less narcogenicity;

c.)possible rectal administration

d.) greater respiratory depression;

29. Allosteric activator of GABA-A receptors:

Choose one answer.

a.) Baclofen

b.) Diazepam

c.) Buspirone

d.) Amizil

30.Antidepressant selective MAO-A inhibitor:

Choose one answer.

a.) Moclobemide

b.) Piracetam

c.) Fluoxetine

d.) Imipramine

e.) Amitriptyline

f.) Caffeine

31.Antidepressant selective serotonin reuptake inhibitor:

Choose one answer.

a.) Piracetam

b.) Fluoxetine

c.) Caffeine

d.) Imipramine

e.) Amitriptyline

32. Does not have an antimanic effect:

Choose one answer.

a.) Haloperidol

b.) Lithium carbonate

c.) Diazepam

d.) Triphthazine

33. Antipsychotic drug from the group of phenothiazine derivatives:

Choose one answer.

a.) Risperidone

b.) Olanzapine

c.) Chlorpromazine

d.) Clozapine

e.) Haloperidol

34.Atypical antipsychotic:

Choose one answer.

a.) Fluorophenazine

b.) Haloperidol

c.) Clozapine

d.) Chlorpromazine

e.) Triphthazine

35. The main psychotropic effect of piracetam:

Choose one answer.

a.) Anxiolytic

b.) Sedative

c.) Mnemotropic

d.) Psychostimulant

36. NSAIDs are characterized by all of the following drug interactions, EXCEPT:

Choose one answer.

a.) codeine weakens the analgesic effect of NA or NSAIDs;

b.) NSAIDs weaken the effect of diuretics and some antihypertensive drugs;

c.) aluminum-containing antacids reduce the bioavailability of NSAIDs

d.) sedatives enhance the analgesic effect of NSAIDs;

37.Day tranquilizer:

Choose one answer.

a.) Phenazepam

b.) Zopiclone

c.) Medazepam

d.) Diazepam

e.) Aminazine

38.Ventricular tachyarrhythmias potential unwanted side effect:

Choose one answer.

a.) Typical antipsychotics

b.) Tricyclic antidepressants

c.) Antidepressants, selective serotonin reuptake inhibitors

d.) Tranquilizers of benzodiazepine derivatives

e.) Atypical antipsychotics

39. Symptoms of acute poisoning with sleeping pills include:

Choose one answer.

a.)excitement, increased blood pressure;

b.) coma, respiratory depression, hypoxia;

c.) increased temperature, increased reflex excitability

40.Which opioid analgesics are contraindicated in myocardial infarction?

Choose one answer.

a.)pentazocine, butorphanol;

b.)morphine, promedol;

c.) fentanyl, nalbuphine

41.What drugs can be used for spasticity of skeletal muscles?

Choose one answer.

a.)strychnine, niketamide (cordiamin), bemegride

b.)baclofen, diazepam, mydocalm;

c.) proserine, galantamine, physostigmine;

42.Which statement regarding acetaminophen (paracetamol) is NOT true?

Choose one answer.

a.) NSAIDs of choice for arthritis

b.) gastrotoxic;

c.) there is no antiplatelet effect;

d.) antipyretic drug of choice for viral infections in children;

43.Which high-potency opioid analgesic is preferable for pain relief during short-term painful procedures/surgeries?

Choose one answer.

a.)morphine;

b.)pentazocine

c.)fentanyl;

d.) promedol;

44. Which drug is classified as an antiepileptic drug?

Choose one answer.

a.)sodium valproate;

b.)levodopa;

c.)cyclodol

45.Which drug is a hypnotic?

Choose one answer.

a.)cyclodol;

b.) zopiclone;

c.)phenytoin (diphenin);

d.)levodopa

46.Which drug is preferable for pain relief in the first stage of labor?

Select one answer.

a.)codeine

b.) metamizole (analgin);

c.)morphine;

d.) trimeperidine (Promedol);

a.) copper preparations

b.) phosphorus

c.) mercury compounds

d.) iron compounds

2. For which medicinal substance is the primary pharmacological reaction due to the effect on DNA transcription processes:

Choose one answer.

a.) insulin;

b.) benzylpenicillin

c.) heparin;

d.) prednisolone;

3. For which drug substance is the primary pharmacological reaction caused by a decrease in the permeability of voltage-gated ion channels:

Choose one answer.

a.) digitoxin;

b.) lidocaine;

c.) ropin;

d.) furosemide

4. For which medicinal substance is the primary pharmacological reaction caused by a decrease in the permeability of mediator-dependent (chemosensitive) ion channels:

Choose one answer.

a.) lidocaine;

b.) pipecuronium

c.) paracetamol;

d.) verapamil;

5. For which medicinal substance is the primary pharmacological reaction due to inhibition of enzyme activity:

Choose one answer.

a.) lidocaine;

b.) adrenaline;

c.) proserin

d.) atropine;

6. For which medicinal substance is the primary pharmacological reaction due to inhibition of the process of facilitated diffusion:

Choose one answer.

a.) adrenaline;

b.) dichlorothiazide.

c.) digoxin;

d.) diazepam;

7. To reduce the concentration of poison in the blood and tissues, use:

Choose one answer.

a.) laxatives

b.) chemical antidotes

c.) adsorbents

d.) functional antidotes

8. To remove unabsorbed poison from the stomach, the latter is washed with water with the addition of:

Choose one answer.

a.) atropine solution

b.) sodium sulfate

c.) methylthioninium chloride (methylene blue)

d.) activated carbon

9. Complexones include:

Choose one answer.

a.) pentacin

b.) naloxone

c.) sodium thiosulfate

d.) pentamine

10.What means are used to stimulate the respiratory center:

Choose one answer.

a.) niketamide (cordiamin); bemegrid; sulfocamphocaine;

b.) morphine; fentanyl; trimeperidine (promedol)

c.) epinephrine (adrenaline); phenylephrine (mesatone); norepinephrine (norepinephrine)

d.) drotoverine (no-spa); metacin; papaverine;

11. Sodium thiosulfate forms low-toxic thiocyanate compounds in case of poisoning:

Choose one answer.

a.) heroin

b.) cyanides

c.) atropine

d.) cardiac glycosides

12.The main goals of treatment of acute poisoning are all except:

Choose one answer.

a.) reducing the concentration of poison in the blood and tissues

b.) reducing further absorption of poison

c.) normalization of the functions of vital organs and systems

d.) slowing down the metabolism of poison

13.The functional antidote of morphine is:

Choose one answer.

a.) diphenhydramine (diphenhydramine)

b.) atropine

c.) naloxone

d.) bemegrid

14. The chemical antidote for Heparin overdose is:

Choose one answer.

a.) phytomenadione

b.) calcium chloride

c.) protamine sulfate

d.) dimercaprol (unithiol)

15. Ethyl alcohol changes the metabolism of poison during poisoning:

Choose one answer.

a.) methyl alcohol

b.) atropine

c.) morphine

d.) arsenic preparations


@ General pharmacology

1. Pharmacodynamics studies

a) Mechanism of action of drugs *

b) Localization of the effect *

c) The effect of the drug on the functions of organs and tissues *

d) Binding of drugs to blood proteins

2. Pharmacogenetic diseases include

a) Malignant hyperthermia *

b) Malignant hypotension

c) Malignant hypertension

3. Fate of the drug in the body:

a) Biotransformation *

b) Use as an energy material *

c) Use as a plastic material *

4. Drug addiction manifests itself

a) Withdrawal syndrome *

b) Presence of abstinence *

c) Tolerance

5. Features of drug therapy in children include

a) Good absorption of drugs from the skin *

b) Poor absorption from the skin

c) Insufficient metabolism of many drugs in the liver *

6. The embryotoxic effect of drugs is realized in the following stages of pregnancy

a) In the first three weeks *

b) After 8 weeks

c) From 4 to 8 weeks

7. Features of enteral drug administration include

a) Digestion affects the absorption process *

b) Possibility of presystemic elimination due to first passage through the liver *

c) Bioavailability is increased compared to parenteral administration of the same drug

8. Selectivity of action is a valuable property of the drug due to

a) Effects on most organs and tissues

b) Minimal side effects *

c) Actions only on a specific organ, tissue or function *

9. Pharmacokinetics studies:

a) The effect of the drug on the functions of organs and tissues

b) Drug distributions *

c) Binding of drugs to blood proteins *

d) Eliminations *

e) Transformations *

10. Bioavailability of a drug is:

a) the amount of drug entering the biophase relative to the administered dose

b) the amount of unchanged substance in the blood plasma relative to the administered dose as a percentage *

c) Drug dose bound to blood proteins

11. Biotransformation can lead to such effects as:

a) Reduced excretion from the body

b) Increased biological activity *

c) Transformation into a water-soluble substance *

d) Reduced biological activity *

12. Repeated use of drugs can lead to:

a) Strengthening the pharmacological effect *

b) Drug addiction *

c) Weakening of the pharmacological effect *

13. Intravenous administration of drugs has the following features:

a) No adsorption *

b) Bioavailability 100%*

c) Controllability of drug dosage by adjusting the rate of administration *



d) Strengthening presystemic elimination

14. Presystemic elimination is:

a) Loss of part of the drug during absorption and first passage through the liver *

b) Biotransformation of drugs in the body

c) Excretion of drugs in urine

15. Bioavailability of a drug is:

a) Part of the dose introduced into the body that has undergone biotransformation

b) Part of the dose introduced into the body that reaches the diseased organ

c) Part of the dose introduced into the body that reached the systemic bloodstream unchanged or in the form of active metabolites *

d) Part of the dose introduced into the body that reaches the brain

16. For which drugs is biotransformation in the liver most significant?

a) Hydrophilic

b) Lipophilic *

c) Gaseous

17. How will the excretion of weak acids in urine change when the pH of primary urine shifts to the alkaline side?

a) Will increase *

b) Will decrease

c) Will not change

18. Receptors belonging to the class of G-protein-associated (“snake”) receptors:

a) N-cholinergic receptors

b) GABA receptors

c) Adrenergic receptors *

d) Insulin receptors

19. The most significant result of the biotransformation of medicinal substances in the body is:

a) Reducing the hydrophilicity of drugs

b) Increased lipophilicity of drugs

c) Increased excretion of drugs by the liver

d) Increased excretion of drugs by the kidneys *

20. Which of the following drugs is a clinically significant inducer of microsomal liver enzymes:

a) Phenobarbital *

b) Cimetidine

c) Erythromycin

d) Succinylcholine

@ Local anesthetics

1. Local anesthetics of the ether type include:

a) Dicaine *

b) Trimecaine

c) Novocaine *

d) Lidocaine

2. The mechanism of action of local anesthetics is associated with:

a) Closure of potassium channels

b) Closing calcium channels

c) Closing sodium channels *

3. Signs of local anesthetic toxicity include:

a) Increased blood pressure

b) Anxiety *

c) Convulsions *

d) Tremor *

a) Validol has a direct stimulating effect on myocardial function

b) Activated carbon enhances the pharmacological effect of bismuth nitrate for gastric ulcers

c) Mustard plasters are irritating to the skin and can change the function of internal organs *

5. Indicate the correct statement:

a) Allergic reactions to local anesthetics are rare *

b) Allergic reactions to local anesthetics are not observed

c) Amide drugs are more allergenic than ester drugs

6. The maximum antiarrhythmic effect is expressed in:

a) Dicaina

b) Lidocaine *

c) Novocaine

7. Enveloping agents have the following effects:

a) Form a protective layer of colloids *

b) Coagulate proteins

c) Adsorb toxic substances

d) Protect sensitive endings from irritation *

8. If signs of toxicity of the local anesthetic appear, you must:

a) Stop administering the drug *

b) Administer an anticonvulsant *

c) Continue administering the drug

d) Rinse the stomach

9. The reason for the cessation of the local anesthetic effect is:

a) Removal from the body

b) Resorption of the drug from the injection site *

c) Destruction of the drug

10. Specify the longest-acting anesthetic:

a) Lidocaine

b) Trimecaine

c) Novocaine

d) Bupivacaine *

11. Local anesthetics of the amide type include:

a) Novocaine

b) Trimecaine *

c) Lidocaine *

d) Dicaine

12. Local anesthetics can block the conduction of electrical impulses:

a) Only along sensory nerve fibers

b) Only along sensory and vegetative fibers

c) Only along sensory and motor fibers

d) Along any nerve fibers *

@ General anesthetics

1. Modern anesthesia includes the following components:

a) Loss of consciousness *

b) Analgesia and suppression of autonomic reactions *

c) Muscle relaxation *

d) Controlled hypotension

2. Select the correct statement regarding nitrous oxide:

a) It is an effective anesthetic and is used as mononarcosis

b) Does not undergo biotransformation in the liver *

c) Does not have analgesic properties

d) Rarely used in modern surgery

3. The following inhalational anesthetics are explosive:

b) Ftorotan

c) Nitrous oxide

4. In modern anesthesia, switching off consciousness is achieved by:

a) Non-inhalational anesthetics *

b) Inhalational anesthetics *

c) Muscle relaxants

5. Widely used inhalational anesthetics are:

a) Ftorotan *

b) Nitrous oxide *

c) Cyclopropane

6. The ether has the following pharmacological effects:

a) Reduces blood pressure

b) Myorelaxation*

c) Analgesic *

7. Ftorotan has the following pharmacological effects:

a) Causes tachycardia

b) Reduces myocardial contractility *

c) Sensitizes the myocardium to adrenaline *

d) Reduces blood pressure *

8. Side effects of ftorotane include:

a) Kidney failure

b) Chemical hepatitis *

c) Arterial hypotension *

9. Name the correct statement regarding ethyl alcohol:

a) The stage of excitation is associated with stimulation of the centers of the cerebral cortex

b) Alcohol can cause mental and physical dependence *

c) Ethyl alcohol is prescribed to prevent hypothermia

10. Advantages of non-inhalational anesthetics include:

a) Low controllability of anesthesia

b) Gentle introduction to anesthesia *

c) Absence of occupational hazard *

d) Possibility of quick induction of anesthesia *

11. Nitrous oxide has the following effects:

a) Depresses breathing

b) Analgesia *

c) Weak anesthetic effect *

d) Myorelaxation

12. Disadvantages of mononarcosis include:

a) Circulatory depression *

b) Prolonged awakening *

c) Respiratory depression *

13. Side effect of ketamine:

a) Arterial hypotension

b) Bronchospasm

c) Post-anesthesia psychosis *

14. Thiopental has the following pharmacological effects:

a) Causes analgesia

b) Reduces blood pressure *

c) Depresses breathing *

d) Inhibits myocardial contractility *

15. Side effect of sombrevin:

a) Chemical hepatitis

b) Histaminogenic effect *

c) Hypertensive crisis

16. Choose the correct statements:

a) Midazolam can be used for premedication *

b) Midazolam has a muscle relaxant effect *

c) Midazolam can be administered intramuscularly *

17. The use of fluorotane is indicated in patients with concomitant pathologies:

a) Hypertensive disease *

b) Coronary heart disease *

c) Liver diseases

d) Bronchial asthma *

18. Indicate the correct statement:

a) Ethyl alcohol is a central nervous system stimulant

b) Ethyl alcohol is a food product

c) Ethanol is an external antiseptic*

19. Property of an inhalational anesthetic that determines the rate of induction of anesthesia:

a) Solubility in fats

b) Solubility in blood *

c) Specific gravity

@ M-cholinergic drugs

1. The mediator in cholinergic synapses is:

a) Carbacholine

b) Acetylcholine *

c) Serotonin

2. M-anticholinergics include:

a) Prozerin

b) Hygronium

c) Atropine *

3. Pharmacological effects of atropine on the eye:

b) Spasm of accommodation

c) Paralysis of accommodation *

d) Mydriasis *

4. Main pharmacological effects of acetylcholine:

a) Increased bronchial tone *

b) Bradycardia *

c) Increased sphincter tone

5. Anticholinesterase drugs include:

a) Armin *

b) Atropine

c) Neostigmine *

d) Galangamine *

6. Indications for the use of proserin are:

a) Decurarization *

b) Myasthenia *

c) Intestinal paresis *

d) Tachyarrhythmia

7. Pharmacological effects of atropine include:

a) Bradycardia

b) Tachycardia *

c) Suppression of salivation*

8. Signs of anticholinesterase drug poisoning include:

a) Tachycardia

b) Bradycardia *

c) Bronchospasm *

d) Salivation*

9. Indicate the correct statements:

a) Ipratropium bromide is used to relieve attacks of bronchial asthma

b) Atropine is a one-sided antagonist of acetylcholine *

c) Scopolamine has an antiemetic effect*

d) In case of atropine overdose, cholinesterase reactivators are used

10. Indications for the use of atropine:

a) Renal colic *

b) As part of premedication *

c) Glaucoma

11. Pharmacological effects of pilocarpine on the eye include:

b) Paralysis of accommodation

c) Mydriasis

d) Decrease in intraocular pressure *

12. Signs of an atropine overdose include:

a) Dry mucous membranes *

b) Tachycardia *

d) Mydriasis *

13. Antidotes for poisoning with anticholinesterase drugs are:

a) Galantamine

b) Carbacholine

c) Atropine *

d) Cholinesterase reagents *

14. For the treatment of glaucoma use:

a) Armin *

b) Atropine

c) Pilocarpine *

15. Indications for the use of anticholinesterase drugs:

a) Decurarization *

b) Treatment of glaucoma *

c) Treatment of peptic ulcer.

16. The following types of cholinergic receptors are distinguished:

a) Muscarinics *

b) Dopamine

c) Serotonin

d) Nicotine *

17. Atropine eliminates bradycardia and AV block because:

a) Reduces the influence of the vagus nerve on the heart *

b) Increases the activity of the sympathetic nervous system

c) Blocks slow calcium channels and reduces myocardial contractility

d) Blocks potassium channels and slows down the rate of repolarization

18. Contraindications to the use of atropine are:

a) Renal colic

b) Gastric ulcer

c) Acute myocarditis

d) Glaucoma *

19. Indications for the use of atropine as an emergency treatment are:

a) Anaphylactic shock

b) AV block*

c) Hypoglycemic coma

d) Hypertensive crisis

20. An undesirable effect when using atropine is:

a) Dry mouth *

b) Bronchospasm

c) Bradycardia

d) Orthostatic hypotension

@ N-cholinergic drugs

1. Short-term ganglion blockers include:

a) Pentamin

b) Hygronium *

c) Pahikarpin

2. Specify a depolarizing type muscle relaxant:

a) Succinylcholine *

b) Tubocurarine

c) Ardoin

d) Tracrium

3. The differences between non-polarizing muscle relaxants and depolarizing ones are:

a) Less deep muscle relaxation

b) Absence of fasciculations *

c) Shorter duration of action

4. Who has the right to work with muscle relaxants:

a) Any doctor

b) Surgeon

c) Anesthesiologist *

a) Cytisine and lobeline are used to treat nicotine addiction *

b) In small doses, nicotine excites H-cholinergic receptors *

c) In large doses, nicotine blocks H-cholinergic receptors *

6. Complications of using succinylcholine include:

a) Muscle pain *

b) Excessive duration of action *

c) Malignant hyperthermia *

d) Arterial hypertension

7. N-cholinomimetics have the following effects:

a) Stimulation of breathing *

b) Sympathetic stimulation *

c) Parasympathetic stimulation *

8. The antidote of a muscle relaxant of non-depolarizing type of action is:

a) N-cholinomimetic

b) M-cholinomimetic

c) anticholinesterase agent *

9. Effect of ganglion blockers:

a) Reduce blood pressure *

b) Increase salivation

c) Cause miosis

10. N-cholinomimetics include:

a) Cytisine *

b) Prozerin

d) Lobelin *

11. Specify non-polarizing muscle relaxants:

a) Tubocurarine *

b) Tracrium *

c) Ardoin *

d) Succinylcholine

12. When using ganglion blockers, complications are observed:

a) Visual impairment *

b) Arterial hypotension *

c) Urinary retention *

d) Postural reactions *

13. Succinylcholine causes:

a) Depolarization of the postsynaptic membrane *

b) Short-term muscle relaxation *

c) Depolarization of the presynaptic membrane

a) Depolarizing type

b) Non-polarizing type *

15. Indications for the use of ganglion blockers:

a) Peptic ulcer

b) Cardiogenic shock

c) Acute left ventricular failure *

d) Hypertensive crisis type I*

a) A combination of atropine and proserine *

b) Atropine

c) Prozerin

17. Indications for the use of muscle relaxants:

a) For tracheal intubation *

b) Switching off spontaneous breathing during mechanical ventilation *

c) Achieving muscle relaxation during surgery *

18. Long-acting ganglion blockers:

a) Used to treat hypertension

b) Used to treat hypertensive crisis

c) Not used*

@Adrenergic agents

1.Norepinephrine has the following effects:

a) Reduces blood pressure

b) Stimulates beta 1 adrenergic receptors *

c) Causes vasospasm *

d) Stimulates alpha receptors *

2. Adrenergic stimulants of indirect action include:

a) Ephedrine *

b) Norepinephrine

c) Mezaton

d) Adrenaline

3. Contraindications to the use of adrenaline:

a) Bronchospasm

b) Arterial hypertension *

4. The mediator in the sympathetic nervous system is:

a) Adrenaline

b) Ephedrine

c) Norepinephrine *

d) Isoproterenol

5. Indicate the correct statements:

a) Norepinephrine acts when administered intravenously for 1 minute *

b) Adrenaline acts when administered intravenously for 5 minutes *

c) Adrenaline acts when administered intravenously for 1 minute

d) Norepinephrine acts when administered intravenously for 5 minutes

6. Adrenaline is:

a) Pure alpha adrenergic stimulant

b) Pure beta-adrenergic stimulant

c) Mixed alpha and beta adrenergic agonist *

7. Alpha blockers are used for:

a) Increased blood pressure

b) Reducing vascular spasm *

c) Decrease in blood pressure *

8. Selective beta 2 adrenergic agonists include:

a) Isoprotenerol

b) Salbutamol *

c) Naphthyzin

9. Stimulation of beta 1 - adrenergic receptors causes:

a) Decreased renin release

b) Increased heart automaticity *

c) Increased myocardial contractility *

d) Increased myocardial excitability *

10. Indications for the use of adrenaline:

a) As an addition to local anesthetic *

b) Treatment of bronchial asthma attacks *

c) Treatment of anaphylactic shock *

11. Stimulation of beta 2 adrenergic receptors causes:

a) Dilation of the bronchi *

b) Vasodilation *

c) Vasoconstriction

d) Narrowing of the bronchi

12. Mesatone is:

a) Pure alpha adrenergic stimulant *

b) Mixed alpha and beta adrenergic stimulant

c) Mainly alpha-adrenergic stimulant

13. Beta 1 - adrenergic blockers have the following effects:

a) Reduce heart rate *

b) Stimulate the release of renin by the kidneys

c) Reduce myocardial contractility *

14. Beta blockers are used to treat:

a) Arrhythmia *

b) Arterial hypertension *

d) Bronchial asthma

15. Simulation of a-adrenergic receptors causes:

a) Vasoconstriction *

b) Vasodilation

c) Narrowing of the bronchi

d) Dilation of the bronchi

16. Alpha adrenergic blockers include:

a) Prazosin *

b) Tropaphen *

c) Propranolol

d) Phentolamine *

17. To treat attacks of bronchial asthma use:

a) Fenoterol *

b) Salbutamol *

c) Propranolol

18. Drugs that primarily block β 1 receptors include:

a) Labetalol

b) Atenolol *

c) Pindolol

19. An undesirable effect when using adrenaline is:

a) Bradyarrhythmia

b) Arterial hypertension *

c) Hypoglycemia

d) Increased intraocular pressure

20. Indications for the use of adrenaline as an emergency remedy are:

a) Anaphylactic shock *

b) Pulmonary edema

c) Hypertensive crisis

d) Paroxysmal ventricular tachycardia

@ Analgesics

1. An opioid receptor antagonist is:

a) Tramal

b) Naloxone *

c) Morphine

d) Promedol

2. Side effects of acetylsalicylic acid:

a) Mental dependence

b) Bleeding *

c) Ulcerogenic effect *

d) Reye's syndrome *

a) The duration of action of fentanyl is 2 hours

b) The duration of action of fentanyl is 20-30 minutes *

c) Opioids cause respiratory depression *

4. The analgesic effect of narcotic analgesics is associated with:

a) Increasing the threshold of pain sensitivity *

b) Inhibition of pain impulses *

c) Decreased emotional response to pain *

5. Addiction to narcotic analgesics is characterized by:

a) Intolerance to the drug

b) The need to reduce the dose of the drug to achieve an effect

c) The need to increase the dose of the drug to achieve an effect *

6. Morphine has the following effects:

b) Causes bronchoconstriction *

c) Causes vomiting *

d) Suppresses cough *

7.Indicate the correct statements:

a) Naloxone eliminates mental and physical dependence on opioids

b) Naloxone reduces respiratory depression in opioid poisoning *

c) Naloxone causes the development of withdrawal symptoms in drug addicts *

8.Indicate the correct statements:

a) The duration of treatment with ketorolac is limited to 5 days *

b) Metamizole (analgin) can cause agranulocytosis *

c) Acetylsalicylic acid in children can provoke acute liver failure and encephalopathy *

9.Main pharmacological effects of analgesics-antipyretics:

a) Antihistamine

b) Anti-inflammatory*

c) Anti-aggregation*

d) Analgesic*

e) Antipyretic *

10. Mechanism of action of analgesics-antipyretics:

a) COX inhibition *

b) Activation of the antinociceptive system

c) Reducing the formation of endogenous pyrogens *

11. The drug of choice for the treatment of opiate poisoning is

a) Caffeine

b) Promedol

c) Naloxone *

d) Fentanyl

12. Indications for prescribing morphine are:

a) Prolonged constipation (constipation)

b) Uncontrollable vomiting

c) Tachycardia

d) Pulmonary edema *

13. A drug that is a specific antagonist of morphine:

a) Naloxone *

b) Diazepam

c) Flumazenil

d) Tramadol

@ Sleeping pills, anticonvulsants

1. Long-term use of barbiturates leads to:

a) Habituation *

b) Dependencies *

c) Induction of microsomal enzymes of hepatocytes *

d) Suppression of the activity of microsomal liver enzymes

2. Barbiturates in hypnotic doses cause:

a) Light sedation

b) Deep sleep *

c) Anticonvulsant effect *

3. Indicate the correct statements:

a) Midosalam is a short-acting benzodiazepine *

b) Diazepam is used for status epilepticus *

c) Reladorm - a combination drug containing barbiturate and BDP *

4. Barbiturate poisoning is accompanied by:

a) Prolonged unconsciousness *

b) Short-term unconsciousness

c) Development of hallucinations

5. The following antiepileptic drugs do not cause drowsiness:

a) Phenobarbital

b) Ethosuximide *

c) Difenin *

6. Pharmacological effects of barbiturates include:

a) Decreased psychomotor reactions *

b) Anticonvulsant effect *

c) Antiparkinsonian effect

7. The following drugs have an anticonvulsant effect:

a) Analeptics

b) Benzodiazepines *

c) Barbiturates *

a) Barbiturates should not be used in the treatment of insomnia *

b) Barbiturates are the drugs of choice in the treatment of insomnia

c) Barbiturates are used in the treatment of epilepsy *

9.Barbiturate poisoning is treated:

a) Forced diuresis *

b) Artificial ventilation *

c) Cardiac stimulants

a) Barbiturates alter the natural structure of sleep *

b) Barbiturates cause sleep close to physiological

c) After using barbiturates, there is difficulty waking up *

d) Barbiturates can cause addiction *

a) Antiepileptic drugs block neuromuscular transmission

b) Diphenin inhibits the transmission of impulses to motor neurons of the spinal cord *

c) Ethosuximide is the drug of choice for petit mal seizures *

12. The following drugs are used in the treatment of parkinsonism:

a) Inhibiting dopamine receptors

b) Anticholinergics *

c) Dopamine receptor stimulants *

13. To relieve status epilepticus, the following are used:

a) IV administration of phenobarbital sodium *

b) Thiopental anesthesia *

c) IV administration of diazepam *

14. Antiparkinsonian drugs include:

a) Levamisole

b) Midantan *

c) Cyclodol *

d) Levodopa *

15. Features of benzodiazepines as hypnotics include:

a) Sleep close to physiological *

b) Psychomotor retardation *

c) Withdrawal syndrome *

16. The drug of choice for preventing grand mal seizures is:

a) Sodium valproate *

b) Ethosuximide

c) Thiopental

@ Psychotropic drugs of depressant type

1. The main pharmacological effects of antipsychotics are:

a) Antipsychotic *

b) Psychostimulating

c) Antiemetic *

2. Select indications for prescribing antipsychotics:

a) Acute psychosis *

b) Treatment of parkinsonism

c) Treatment of persistent vomiting *

d) As part of neuroleptanalgesia *

3. Antipsychotic effect is observed in:

a) Bromine preparations

b) Aminazine *

c) Droperidol *

4. The antipsychotic effect of neuroleptics is associated with:

a) With blockade of dopaminergic receptors *

b) With limited influence of the frontal lobes on the limbic system *

5.Side effects of aminazine include:

a) Arterial hypertension

b) Hepatotoxicity *

c) Parkinsonism *

6. List the drugs for the treatment and prevention of manic state in manic-depressive psychosis:

a) Benzodiazepines

b) Neuroleptics *

c) Lithium preparations *

7. Sedatives include:

a) Eleutherococcus extract

b) Corvalol *

c) Valerian extract *

d) Bromcamphor *

8. Indicate the correct statement:

a) Lithium drugs for mania have a rapid effect

b) Lithium drugs for mania have a delayed effect *

9. Benzodiazepines have the following pharmacological effects:

a) Sedative *

b) Sleeping pills *

c) Central muscle relaxant *

d) Anticonvulsant*

10. Benzodiazepines can be used:

a) As part of premedication*

b) As anticonvulsants *

c) To relieve stress before a performance

11. Benzodiazepines can cause:

a) Analgesia

b) Anxiety

c) Indifference *

d) Antiphobic effect *

12. The mechanism of action of benzodiazepines is associated with:

a) Increased activity of GABAergic structures of the brain *

b) Stimulation of benzodiazepine receptors *

13. Select a “daytime” tranquilizer:

a) Nozepam *

b) Diazepam

c) Phenazepam

d) Nitrazepam

14. Name the antidote for benzodiazepine overdose:

a) Naloxone

b) Flumazenil *

c) Atropine

d) Acetylcysteine

a) Acute and severe manic states are treated with lithium salts

b) Acute and severe manic state is treated with aminazine *

c) To prevent a manic state in MDP, lithium salts are used *

16. Check the correct statements:

a) Bromides are well absorbed from the gastrointestinal tract *

b) An overdose of bromine preparations may cause seizures

c) Bromism is characterized by skin rashes, rhinitis, conjunctivitis *

17. Side effects of anxiolytics may include:

a) Drug addiction *

b) Drowsiness *

c) Impaired psychomotor reactions *

d) Psychosis

18. Drug used to reverse the effects of benzodiazepines:

a) Naloxone

b) Naltrexone

c) Diazepam

d) Flumazenil *

e) Levodopa

19. Drug used for status epilepticus:

a) Sodium valproate

b) Diazepam *

c) Carbamazepine

d) Ethosuximide

20. A drug with an antiemetic effect:

a) Diazepam

b) Haloperidol *

c) Caffeine

d) Piracetam

e) Phenobarbital

21. Drug used to create neuroleptanalgesia:

a) Amitriptyline

b) Caffeine

c) Levodopa

d) Diazepam

e) Droperidol *

22. A side effect common to most phenothiazines is:

a) Increased blood pressure

b) Parkinsonism *

c) Suppression of prolactin release

@ Psychotropic drugs of stimulating type of action

1. MAO inhibitors cause the following side effects:

a) Insomnia *

b) Hepatotoxicity *

c) Psychostimulation *

2.Antidepressants include:

a) Caffeine

b) Imizin *

c) Amitriptyline *

d) Fluoxetine *

3. Which of the following properties does piracetam have:

a) Has a beneficial stimulating effect on mental activity *

b) Has an antihypoxic effect*

c) The effect appears after the first dose

4. Psychostimulants are used for the following indications:

a) Acute psychosis

b) Narcolepsy *

c) Stimulation of mental and physical activity during fatigue *

5. MAO inhibitors include:

a) Aminazine

b) Nialamid *

c) Amitriptyline

6. Pharmacological effects of caffeine:

a) Increases mental and physical performance *

b) Enhances the effect of non-narcotic analgesics *

c) Causes cardiac stimulation *

7. Antidepressants (tricyclic compounds) have the following pharmacological effects:

a) Antidepressant *

b) Antipsychotic

c) Indirect stimulation of serotonin and adrenergic receptors in the central nervous system *

8. General tonics (adaptogens) include:

a) Pantocrine *

b) Phenamine

c) Ginseng preparations *

9. Psychostimulants include:

a) Phenamine *

b) Caffeine *

c) Aminalon

d) Pantocrine

10. Nootropic drugs include:

a) Aminalon *

b) Caffeine

c) Nootropil *

d) Cerebrolysin *

11. Piracetam causes the following side effects:

a) Increased irritability *

b) Hepatotoxicity

c) Respiratory depression

d) Sleep disturbance *

12. Pharmacological effects of piracetam include:

a) Awakening effect in comatose states *

b) Improves memory and learning when they are impaired *

c) Increases blood flow in the capillaries of the brain *

13. Analeptics include:

a) Caffeine *

b) Clozapine

c) Cordiamine *

d) Camphor *

14. The use of nootropics is indicated for:

a) Alcohol delirium *

b) Chronic cerebral insufficiency *

c) Psychoorganic disorders of traumatic, toxic and vascular origin *

15.The use of caffeine is indicated in the following cases:

a) For the treatment of enuresis in children *

b) For the treatment of insomnia

c) To stimulate mental activity *

@ Drugs affecting the functions of the digestive system

1. Replacement therapy means include:

a) Pancreatin *

b) Festal *

c) Acidin-pepsin *

2. True hepatoprotectors include:

a) Ademethionine *

c) Essentiale *

3. Drugs that reduce gastric secretion include:

a) Ranitidine *

b) Maalox

c) Omeprazole *

4. Anti-Helicobacter combination pharmacotherapy includes:

a) Antacids

b) Antibiotics *

c) Proton pump inhibitors or H2-histamine blockers *

5. For vomiting caused by cytostatics, it is effective:

a) Scopolamine

b) Tropisetron*

6. Drugs that enhance intestinal motility include:

a) Aceclidine *

b) Prozerin *

c) Bisacodyl *

7. H2-histamine receptors in the stomach are blocked by the following drugs:

a) Famotidine *

b) Nizatidine *

c) Ranitidine *

8. The following have an antiemetic effect:

a) Neuroleptics *

b) Antihistamines *

c) Anticholinergics *

d) Serotonin antagonists *

e) Dopamine antagonists *

9. In case of acute poisoning, the optimal use as a laxative is:

a) Buckthorn extract

b) Magnesium sulfate

c) Sodium sulfate *

10. Indicate the correct statements:

a) Film-forming agents reduce the volume of gastric secretion

b) A proton pump inhibitor blocks the synthesis of hydrochloric acid *

c) Bisacodyl is indicated for acute poisoning

11. Gastroprotectors include:

a) Essentiale

b) Misoprostol *

c) Sucralfate *

12. Indications for the use of H2-histamine receptor blockers:

a) Duodenal ulcer *

b) Hyperacid gastritis *

c) Gastric ulcer *

13. Gastroprotectors have the following effects:

a) Neutralize hydrochloric acid

b) Form a protective coating *

c) Stimulate regeneration *

14. Stimulators of bile formation (choleretics) include:

b) Allohol *

c) Magnesia sulfate

15. Systemic antacid drugs include:

a) Sodium bicarbonate*

b) Almagel

c) Maalox

@ Drugs affecting respiratory function

1. The following have a bronchodilator effect:

a) Alpha-adrenergic receptor agonists

b) Beta 2-adrenergic receptor agonists *

c) Phosphodiesterase inhibitors *

d) Mast cell membrane stabilizers

2. Enzyme preparations include:

a) Acetylcysteine

b) Bromhexine

c) Chymotrypsin *

d) Ribonuclease *

3. Check out the centrally acting antitussives:

a) Codeine *

b) Bromhexine

c) Glaucine *

4. Codeine has the following effects:

a) Analgesic *

b) Antitussive *

c) Causes drug dependence *

d) Stimulates breathing

5. What drugs are suitable for stopping an attack of bronchial asthma

a) Salbutamol *

b) Beclomethasone

c) Adrenaline *

d) Ipratropium bromide

a) Bromhexine stimulates the production of surfactant *

b) Glucocorticoids are used by inhalation to prevent attacks of bronchial asthma *

c) Cromolyn sodium is used in long courses *

7. Which M-anticholinergic drug is preferable for the treatment of bronchial asthma?

a) Atropine

b) Scopolamine

c) Ipratropium bromide *

8. Mast cell membrane stabilizers:

a) Affect the immunological link in the pathogenesis of bronchial asthma *

b) Can stop an attack of bronchial asthma

c) Used to prevent seasonal exacerbations of asthma *

9. The following bronchodilators are used by inhalation:

a) Salbutamol *

b) Salmeterol *

c) Ipratropium bromide *

10. Reflex expectorants include:

a) Thermopsis *

b) Mukaltin *

c) Bromhexine

11. List selective b 2 -adrenergic agonists:

a) Salbutamol *

b) Adrenaline

c) Fenoterol *

12.Indicate the correct statements:

a) Codeine is not prescribed to children under 2 years of age *

b) Reflex expectorants in high doses can cause vomiting *

c) Antitussives are not prescribed for pulmonary hemorrhages *

13. Indicate the correct statements:

a) Inhaled forms of glucocorticoids have minimal systemic effect *

b) Mast cell membrane stabilizers prevent bronchospasm *

c) Theophylline has a small breadth of therapeutic action *

14. Contraindications to the use of antitussives:

a) Hypersecretory bronchitis *

b) Dry non-productive cough

c) Pulmonary hemorrhage *

15. What is characteristic of libexin?

a) Suppresses the cough reflex, acting peripherally *

b) Equal to codeine in effectiveness for coughing

c) Reduces the excitability of sensory nerve endings *

d) Does not cause addiction or drug dependence *

16. Drugs with a mucolytic effect include:

a) Codeine

b) Bromhexine*

c) Cititon

17. Expectorants:

a) Bromhexine*

b) Tusuprex

c) Glaucine

18. Antitussives should be prescribed for:

a) Exudative pleurisy*

b) Non-productive cough*

c) Oncological diseases of the respiratory system*

@Medicines used in the treatment of heart failure

1. Indications for the use of osmotic diuretics are:

a) Pulmonary edema

b) Forced diuresis *

c) Brain edema *

d) Hypertensive crisis

2. Carbonic anhydrase inhibitors are characterized by:

a) Weak diuretic effect *

b) Rapid development of resistance *

c) Decreased production of intraocular fluid *

3. Note the features of the action of furosemide when administered intravenously:

a) Slow development of the effect

b) Short-term effect (2-4 hours) *

c) High diuretic activity *

4. Select the features of the action of dichlorothiazide:

a) Duration of action 1-2 hours

b) Duration of action 8-12 hours *

c) Lowers blood pressure in arterial hypertension *

5. Specify a diuretic that affects the ascending limb of the loop of Henle:

a) Spironolactone

b) Ethacrynic acid *

c) Dichlorothiazide

d) Diacarb

6. For the treatment of chronic heart failure use:

b) Diuretics *

c) b-blockers *

d) Ganglioblockers

e) Cardiac glycosides *

7. Extrarenal effects of furosemide include:

a) Vasodilation of arterioles *

b) Vasodilation of venules *

c) Reduced preload *

d) Reduced afterload *

8 Complications with loop diuretics include:

a) Hypokalemia *

b) Hyponatremia *

c) Hypovolemia *

d) Metabolic acidosis

9. Specify the side effects of ACE inhibitors:

a) Dry cough *

b) Excessive hypotension *

c) Mydriasis

10. Treatment of intoxication with cardiac glycosides includes:

a) Forced diuresis

b) Use of sera containing antibodies to FH *

c) Drug withdrawal *

11. Potassium-sparing diuretics have the following effect:

a) Strong diuretic

b) Reduce potassium loss *

c) Sharply reduce blood pressure

12. Indications for prescribing loop diuretics are:

a) Chronic heart failure *

b) Pulmonary edema *

c) Intoxication with cardiac glycosides

13. Cardiac glycosides are used:

a) For the treatment of ventricular arrhythmias

b) For the treatment of bradyarrhythmias

c) For the treatment of CHF *

14. The mechanism of action of furosemide is associated with:

a) Exchange of sodium for hydrogen ions

b) Inhibition of Na + -K + -2Cl - cotransporter *

c) An increase in osmotic pressure in the tubule

15. First-line drugs for chronic heart failure are:

a) ACE inhibitors *

b) Diuretics

c) Beta-blockers

d) Cardiac glycosides

16. Signs of intoxication with cardiac glycosides include:

a) Extrasystole *

b) Xanthopsia *

c) Bradycardia *

d) AV block *

17. Potassium-sparing diuretics include:

a) Spironolactone *

b) Furosemide

c) Triamterene *

d) Amiloride *

18. The mechanism of the diuretic action of spironolactone is due to:

a) Blockade of carbonic anhydrase

b) Increased glomerular filtration

c) Inhibition of aldosterone synthesis

d) Elimination of the influence of aldosterone on the renal tubules *

19. Specify a diuretic that acts primarily in the area of ​​the loop of Henle:

a) Spironolactone

b) Mannitol

c) Furosemide *

d) Indapamide

20. Cardiac glycosides include:

a) Dopamine

b) Digitoxin *

c) Adrenaline

@ Antiarrhythmic drugs

1. Quinidine has:

a) Vagolytic action *

b) alpha-adrenergic blocking effect *

c) Direct myolytic effect *

2. Quinidine has the following effects:

a) Reduces the rate of depolarization *

b) Membrane stabilizing *

c) Reduces myocardial contractility *

3. Indications for the use of lidocaine:

a) Supraventricular tachycardia

b) Ventricular extrasystole *

c) Morgagni-Adam-Stokes syndrome

4. Verapamil is used for:

a) Supraventricular arrhythmias *

b) Ventricular arrhythmias

c) Weakness of the sinus node

d) Ventricular tachycardia

5. Verapamil is contraindicated in:

a) Arterial hypotension *

b) Atrial fibrillation

c) Atrial flutter

6. Drug dependence can occur when using:

a) Quinidine

b) Propranolol *

c) Diphenina

d) Ephedrine *

7. Toxic effects of amiodarone include:

a) Arterial hypotension *

b) Bradycardia *

c) Pulmonary damage *

d) Corneal pigmentation *

8. For bradycardia the following are used:

a) Atropine *

b) Ephedrine *

c) Izadrin *

9. Class I B antiarrhythmics include:

a) Verapamil

b) Lidocaine *

c) Amiodarone

10. Class I A antiarrhythmics include:

a) Quinidine *

b) Novocainamide *

c) Novocaine

d) Lidocaine

11. Novocainamide is used in the treatment of:

a) Ventricular arrhythmias *

b) Supraventricular arrhythmias*

c) Bradyarrhythmia

12. Toxic effects of lidocaine include:

a) Dizziness and muscle twitching *

b) Convulsions *

c) Gastrointestinal disorders

13. Lidocaine:

a) Effective for ventricular arrhythmias *

b) Effective for supraventricular arrhythmias

c) Reduces myocardial contractility

14. Class 3 antiarrhythmics includes:

a) Atenolol

b) Amiodarone *

c) Verapamil

15. Propranolol has the following effects:

a) Slows down AV conduction *

b) Suppresses ectopic foci *

c) Reduces the automaticity of the sinus node *

16. Class 4 antiarrhythmic drugs include:

a) Propranolol

b) Verapamil *

c) Amiodarone

d) Ethmozin

17. Beta-blockers are contraindicated in:

a) Sinus tachycardia

b) Sinus bradycardia *

c) AV block *

d) Severe heart failure *

18. Amiodarone:

a) Causes peripheral vasodilation *

b) Used for ventricular arrhythmias *

c) Causes a decrease in myocardial contractility *

d) Used for supraventricular arrhythmias *

19. Class 1A antiarrhythmic drugs include:

a) Verapamil

b) Lidocaine

c) Quinidine *

20. To relieve ventricular arrhythmia in acute myocardial infarction, the following is used:

a) Lidocaine *

b) Atropine

c) Verapamil

@ Antihypertensive drugs

1. Side effects of clonidine include:

a) Euphoria

b) Tachycardia

c) Withdrawal syndrome *

2. When using which antihypertensive drug may the development of orthostatic hypotension be possible:

a) Dichlorothiazide

b) Propranolol

c) Prazosin *

3. The following have a sympatholytic effect:

a) Octadine *

b) Reserpine *

c) Verapamil

4. Direct vasodilators include:

a) Clonidine

b) Sodium nitroprusside *

c) Pentamin

5. The following calcium channel blockers are used as antihypertensive drugs:

a) Nifedipine *

b) Captopril

c) Diltiazem *

d) Amlodipine *

6. The hypotensive effect of hypothiazide is achieved through:

a) Minutes

c) Weeks *

7. Indicate the correct statements:

a) Captopril is an indirect vasodilator *

b) Captopril is prescribed once a day

c) The usual single dose of captopril is 25 mg *

8. To relieve a hypertensive crisis use:

a) Nifedipine *

b) Clonidine *

c) Captopril *

d) Atenolol

9. Specify drugs that are blockers of slow calcium channels:

a) Nifedipine and procainamide

b) Verapamil and nifedipine *

c) Amiodarone and lidocaine

10. Indicate the correct statements:

a) Capozide is an alternative to the 2nd stage of treatment of hypertension*

b) Captopril can be prescribed to pregnant and nursing mothers

c) Captopril is used in the treatment of heart failure *

11. Sodium nitroprusside has the following effects:

a) Reducing preload *

b) Vasodilation of arterioles *

c) Reduced afterload *

d) Vasodilation of venules *

12. Orthostatic disorders are typical for the following antihypertensive drugs:

a) Prazosin *

b) Reserpine

c) Octadine *

13. Centrally acting alpha-agonists include:

a) Prazosin

b) Clonidine *

c) Dopegit *

d) Moxonidine *

14. For the treatment of chronic arterial hypertension, the following drugs can be used as monotherapy:

a) ACE inhibitors *

b) Rauwolfia preparations

c) Thiazide diuretics *

15. Captopril:

a) Blocks β-adrenergic receptors

b) Inhibits angiotensin-converting enzyme *

c) May cause hypertension

d) Used to treat bronchial asthma

16. Verapamil:

a) Blocks ά-adrenergic receptors

b) Blocks sodium channels

c) Has antiarrhythmic and antihypertensive effects *

d) Causes withdrawal syndrome

17. The development of orthostatic hypotension is possible when using:

a) Hydrochlorothiazide

b) Nitroglycerin *

c) Verapamil

18. Identify a drug that has the following mechanism of action: stimulates alpha 2 - adrenergic receptors, reduces the tone of vasomotor centers, has a sedative effect, reduces cardiac output and vascular resistance:

a) Hydralazine

b) Prazosin

c) Clonidine *

d) Propranolol

19. To relieve hypertensive crisis type II, the following is used:

a) Amlodipine

b) Clonidine*

c) Hydrochlorothiazide

@ Antianginal drugs

1. Beta blockers have the following cardiac effects:

a) Positive chronotropic

b) Negative chronotropic *

c) Positive dromotropic

2. Long-acting nitroglycerin preparations include:

a) Nitroglycerin solution 1% oil in capsules

b) Aerosol

c) Nitromast *

3. Indicate the correct statement:

a) Amlodipine is a beta-blocker

b) Amlodipine is a calcium channel blocker *

c) Amlodipine is an alpha-blocker

4. Indicate the correct statement:

a) Trimetazidine is classified as a Ca ++ channel blocker

b) Trimetazidine is prescribed to relieve an anginal attack

c) Trimetazidine is used for course treatment of ischemic heart disease *

5. Ca++ channel blockers, dihydropyridine derivatives:

a) Amlodipine *

b) Verapamil

c) Diltiazem

d) Felodipine*

6. The antianginal effect of nitrovasodilators is associated with:

a) Coronary vasodilation *

b) Reducing preload *

c) Reducing afterload *

7. Verapamil:

a) Causes a decrease in myocardial contractility *

b) Causes increased myocardial contractility

c) Is a Ca++ synergist

d) Is a calcium channel blocker *

8. Complications during treatment with nitrates:

a) Withdrawal syndrome *

b) Headache *

c) Tolerance *

d) Orthostatic collapse *

9. There is no effect of nitroglycerin when taken orally due to:

a) Lack of suction

b) Destruction in the stomach

c) Destruction in the liver *

d) Rapid excretion by the kidneys

10. The mechanism of antianginal action of beta 1 and non-selective beta blockers:

a) Increased oxygen delivery

c) Reducing preload and afterload

11. Which of the following drugs are indicated for angina pectoris?

a) Propranolol *

b) Aceclidine

c) Adrenaline

d) Metoprolol *

12. Ca++ channel blockers, non-dihydropyridine derivatives:

a) Amlodipine

b) Verapamil *

c) Diltiazem *

d) Felodipine

13. The combination of beta blockers and verapamil is unacceptable due to:

a) Increased vasodilation

b) Pharmaceutical incompatibility

c) The likelihood of developing AV block and bradycardia*

14. Indications for the use of nitroglycerin include:

a) Acute attack of angina *

b) Myocardial infarction *

c) Cardiogenic pulmonary edema *

15. Myocardial oxygen demand increases with:

a) Increased myocardial contractility *

b) Decreased myocardial contractility

c) Increased heart rate *

16. The drug of choice for the treatment of an anginal attack is:

a) Nitrosorbide

b) Verapamil

c) Nitroglycerin *

d) Nitromas

17. Beta blockers cause:

a) Vasodilation

b) Decreased myocardial contractility *

c) Bronchodilation

d) Decrease in blood pressure *

18. Nitroglycerin causes:

a) Reducing preload *

b) Reduced myocardial oxygen demand *

c) Reduced afterload *

19. The duration of action of nitroglycerin when taken sublingually is:

a) 5-15 minutes

b) 20-30 minutes *

c) 40-60 minutes

20. In the treatment of chronic ischemic heart disease, the following is used:

a) Isosorbide dinitrate *

b) Propranolol *

c) Nitroglycerin intravenously

21. Side effect of nitrates:

a) Kidney failure

b) Headache *

c) Increased blood pressure

@ Drugs affecting hemostasis

1. Thrombolytics:

a) Cause hemostasis

b) Do not affect the size of the blood clot

c) Lyse the thrombus *

2. Fibrinolysis inhibitors include:

a) Contrikal *

b) Aminocaproic acid *

c) Streptolyasis

3. Antiplatelet agents include:

a) Acetylsalicylic acid *

b) Neodicoumarin

c) Ticlopidine *

4. Antiplatelet agents are used for:

a) Prevention of arterial thrombosis *

b) Prevention of venous thrombosis

c) Prevention of pulmonary embolism

5. Indications for heparin:

b) Pulmonary embolism*

c) Acute myocardial infarction *

6. Side effects of acetylsalicylic acid:

a) Acute gastric erosions *

b) Aspirin asthma *

c) Stomach bleeding *

d) Thrombosis

7. Clinical indications for the use of heparin include:

a) Prevention of phlebothrombosis *

b) Treatment of phlebothrombosis *

c) Treatment of pulmonary embolism *

8. Drugs that increase myometrial tone include:

a) Ergometrine *

b) Ergotamine *

c) Fenoterol

9. Thrombolytics are contraindicated:

a) In the first 3 days after surgery *

b) During labor *

c) For peptic ulcer of the stomach and duodenum *

10. Indications for thrombolytic therapy:

a) Acute arterial thrombosis in the early stages *

b) Acute myocardial infarction *

c) Pulmonary embolism *

11. Drugs that stimulate uterine contractility include:

a) Oxytocin *

b) Pituitrin *

c) Dinoprost *

12. Antiplatelet agents have the following effects:

a) Prevent phlebothrombosis

b) Lyse arterial thrombus

c) Prevent platelet aggregation *

13. Low molecular weight heparin preparations:

a) Have higher availability when administered subcutaneously than regular heparin *

b) Actively lyse the formed thrombus

c) Rarely cause thrombocytopenia *

14. The average therapeutic single dose of heparin is:

a) 2.5-5 thousand units

b) 7.5-10 thousand units *

c) 10.0-12.5 thousand units

15. Medium doses of heparin provide for prolongation of blood clotting time

a) Does not change significantly

b) 2-3 times *

c) 1.5-2 times

16. Contraindications to the use of heparin are:

a) Active bleeding *

b) Pregnancy

c) Upcoming surgery *

d) Hypercoagulation

17. The antiplatelet effect of acetylsalicylic acid is associated with:

a) Inhibition of thromboxane synthetase

b) Stimulation of thromboxane synthetase

c) Inhibition of COX *

d) Inhibition of phosphodiesterase

e) Inhibition of adenylate cyclase

18. Anticoagulants include:

a) Dalteparin sodium and aprotinin

b) Streptokinase and warfarin

c) Warfarin and heparin *

19. To stop bleeding use:

a) Urokinase

b) Streptokinase

c) Warfarin

d) Etamsylate*

20. Heparin:

a) Is an indirect anticoagulant

b) Inactivates thrombin *

c) Increases platelet aggregation

d) Effective when taken orally

21. Antidote for heparin overdose:

a) Protamine sulfate*

b) Vikasol

c) Acetylsalicylic acid

@ Drugs affecting hematopoiesis

1. Cyanocobalamin is used to treat megaloblastic anemia:

a) Enterally

b) Subcutaneously *

c) Intramuscularly *

d) Intravenously *

2. Most antitumor drugs have the following effects on the blood:

a) Anemia *

b) Leukopenia *

c) Thrombocytopenia *

3. Basic requirements for a combination of antitumor drugs:

a) A different mechanism of action for each drug *

b) Different side effects for each drug *

c) The presence of an effect for each drug for a given type of tumor *

4. Cyanocobalamin deficiency leads to:

a) Hypochromic anemia

b) Peripheral paralysis *

c) Renal disorders

5. Leukopoiesis stimulants include:

a) Methyluracil *

b) Pentoxyl *

d) Leucomax *

6. Parenteral iron preparations include:

a) Koamid

b) Ferrum Lek *

c) Ferkoven *

7. The effect of enteral iron supplements may be accompanied by:

a) Nausea *

b) Vomiting *

c) Anorexia *

d) Constipation *

8. Megaloblastic anemia occurs as a result of deficiency:

a) Gland

b) Vitamin B 12 *

c) Cobalt

9. Hormonal antitumor drugs are characterized by the following properties:

a) Have a cytotoxic effect

b) Slow down the growth of tumor cells *

c) Contribute to the restoration of disturbances in the humoral regulation of cells *

10. Enteral iron supplements include:

a) Sorbifer*

b) Ferroplex *

c) Ferkoven

11. Typical side effects of anticancer drugs:

a) Anorexia *

b) Nausea and vomiting *

c) Bone marrow depression *

12. Hypochromic anemia is caused by:

a) Iron deficiency *

b) Folic acid deficiency

c) Vitamin B 12 deficiency

13. Antimetabolites include:

a) Mercaptopurine *

b) Ftorafur *

c) Methotrexate *

d) Myelosan

14. Antineoplastic drugs are usually used in the form of:

a) Monotherapy

b) Sequential drug therapy *

c) Combinations of several drugs *

15. Antidote for an overdose of iron:

a) Desferal*

b) Protamine sulfate

c) Vikasol

@ Hormonal drugs

1. Indications for the use of glucocorticoids:

a) Collagenoses *

b) Bronchial asthma *

c) Acute infections

d) Acute and chronic adrenal insufficiency *

2. Side effects of glucocorticoids include:

a) Suppression of the adrenal cortex *

b) Exacerbation of infection *

c) Hyperglycemia *

d) Osteoporosis *

3. Mineralocorticoids have the following effects:

a) Sodium retention *

b) Increased potassium loss *

c) Water retention *

4. Indications for prescribing insulin are:

a) Diabetes mellitus type 1 *

b) Diabetic coma *

c) As part of a polarizing mixture *

5. One of the main effects of glucocorticoids:

a) Decrease in blood glucose levels

b) Anabolic effect

c) Anti-inflammatory effect *

d) Delay in the body K +

6. Antithyroid effects have:

a) Mercazolil *

b) Radioactive iodine preparations *

c) Triiodothyronine

7. In the treatment of diabetes mellitus they use:

a) Short-acting insulin intravenously

b) Intermediate-acting insulin intravenously

c) Short-acting insulin subcutaneously *

8. Select the correct statements: Anabolic steroids

a) Exhibit an androgenic effect *

b) Cause masculinization in women *

c) Accelerate the process of bone calcification *

9. What metabolic changes are caused by thyroxine when it is overproduced?

a) Enhances protein breakdown *

b) Reduces basal metabolism

c) Helps reduce body weight *

10. Glucocorticoids cause the following effects:

a) Anti-inflammatory*

b) Increase the loss of potassium and calcium *

c) Immunosuppressive *

d) Causes steroid diabetes*

11. Insulin reduces hyperglycemia by:

a) Increase in the number of insulin receptors

b) Movement of glucose into cells *

c) Increased glucose utilization *

12. Hypoglycemic drugs for oral administration are used for:

a) Diabetes mellitus type I

b) Diabetic coma

c) Diabetes mellitus type II *

13. Contraindications to the administration of glucocorticoids:

a) Diabetes mellitus *

b) Gastric ulcer *

c) Osteoporosis *

14. Indicate the correct statements:

a) Biguanides increase the utilization of glucose by cells *

b) Biguanides inhibit the absorption of glucose in the intestine *

15. State the correct statement. Treatment of diabetic coma is carried out:

a) subcutaneous administration of short-acting insulin

b) Intravenous administration of short-acting insulin *

c) Intravenous administration of long-acting insulin.

a) Biguanides act on extrapancreatic factors and have an anorexigenic effect *

b) Combined multiphase contraceptives imitate the normal course of the menstrual cycle *

c) Sulfourea derivatives cannot be combined with insulin

17. Note the side effects of corticotropin

a) Decreased blood pressure

b) Edema *

c) Delay in regeneration processes *

d) Insomnia *

18. For the treatment of diabetic coma the following is used:

a) Prednisolone

b) 40% glucose solution

c) Glucagon

d) Insulin for injection *

19. Side effects of glucocorticoids:

a) Hypoglycemia and tachycardia

b) Cachexia and osteoporosis

c) Osteoporosis and hyperglycemia *

20. Anabolic steroids:

a) Remove Ca ++ from the body

b) Retain Na + and water in the body *

c) Inhibit protein synthesis

21. The drug has an anabolic effect:

a) Triiodothyronine

b) Thyroidin

c) Dexamethasone

d) Insulin *

22. Indications for the use of glucocorticoids are:

a) Osteoporosis

c) Anaphylactic shock *

d) Hyperglycemic coma

@ Vitamin preparations

1. Vitamin deficiency manifests itself as:

a) Exacerbation of chronic infections *

b) Polyhypovitaminosis *

c) Decreased immunity *

2. Vitamin deficiency leads to:

a) Decreased performance *

b) Increased fatigue *

c) Mental depression

3. Indicate the correct statements:

a) Vitamins B and C are water soluble *

b) Vitamin C has antioxidant properties *

c) Vitamin A is necessary for full twilight vision *

4. Indications for the use of multivitamins:

a) Children's age *

b) Arterial hypotension

c) Hard physical labor *

d) Pregnancy and lactation *

5. Vitamin P preparations are:

a) Troxevasin *

b) .Ascorutin *

c) Rutin *

6. In case of an overdose of vitamin D, the following are noted:

a) Soft tissue calcification *

b) Kidney damage *

c) Decalcification of the skeleton *

d) Increased atherosclerotic changes *

7. Indications for vitamin D therapy:

a) Rickets *

b) Osteomalacia and osteopathy *

c) Long-term artificial feeding *

d) Deceleration of bone healing during fractures *

8. Indicate the correct statements:

a) Vitamin C in large doses can cause exacerbation of urolithiasis *

b) Vitamin P protects ascorbic acid from oxidation *

c) Retinoids have a teratogenic effect *

9. Vitamin B1 preparations are used for:

a) For the treatment of neuritis, radiculitis *

b) During alcohol detoxification *

c) To maintain pregnancy

10. Vitamin B2 preparation is:

a) Riboflavin *

b) Pyridoxine

c) Cocarboxylase

11. Indicate the correct statements:

a) Cholestyramine is absorbed in the intestine and disrupts lipid synthesis in the liver

b) Nicotinic acid has hepatoprotective properties *

c) Lovastatin is the most effective drug in the treatment of atherosclerosis *

12. Basic mechanisms of action of anti-atherosclerotic drugs:

a) Inhibition of lipid absorption in the intestine *

b) Activation of lipid destruction

c) Impaired cholesterol formation *

13. The following have antioxidant properties:

a) Vitamin D

b) Vitamin A *

c) Vitamin C *

d) Vitamin E *

14. Vitamin B 6 preparation is:

a) Cyanocobalamin

b) Pyridoxine *

15. Disturbs the absorption of lipids and cholesterol in the intestine:

a) Cholestyramine *

b) Lovastatin

c) Clofibrate

16. Note the side effects associated with an overdose of vitamin A:

a) Headache *

b) Drowsiness *

c) Skin lesions *

17. Indications for therapy with pyridoxine:

a) Treatment of tuberculosis *

b) Intensive antibiotic therapy *

c) Neuritis, radiculitis *

18. Enzyme preparations include:

a) Lidaza *

b) Panzinorm *

c) Cocarboxylase

d) Insulin

19. Specify antioxidant vitamins:

a) Ascorbic acid and tocopherol *

b) Tocopherol and cyanocobalamin

c) Cyanocobalamin and retinol

20. Retinol in doses exceeding the physiological requirement, and its derivatives cause:

a) Teratogenic effect*

b) Hemeralopia

c) Hair loss

21. Specify the antioxidant vitamin:

a) Ascorbic acid*

b) Cyanocobalamin

c) Folic acid

@ Anti-inflammatory drugs

1. Immunosuppressive therapy is necessary:

a) To suppress the tumor process

b) In the treatment of AIDS

c) In the treatment of autoimmune diseases *

2. The main pharmacological effects of NSAIDs include:

a) Antiaggregation effect *

b) Anti-inflammatory effect *

c) Antipyretic effect *

d) Analgesic effect *

3. Immunomodulators include:

a) Cyclosporine

b) Levamisole *

c) Prodigiosan *

d) Timalin *

4. Indications for the use of glucocorticoids:

a) Treatment of status asthmaticus *

b) Treatment of severe infections

c) Rheumatoid arthritis *

5. Antihistamines H 1 are used for:

a) Prevention of anaphylactic reactions *

b) Treatment of itchy dermatoses *

c) For the treatment of asthma

6. Second generation antihistamines

a) Have a prolonged action *

b) Practically do not enter the central nervous system *

c) Inhibit the release of inflammatory mediators from mast cells *

7. The antiallergic effect of glucocorticoids is carried out by:

a) Depression of the function of cellular immunity factors *

b) Inhibition of the interaction of receptors on the surface of mast cells with antibodies *

c) Increased sensitivity of adrenergic receptors to the effect of agonists *

8. Main effects of diphenhydramine:

a) Reduces the effects of histamine *

b) Antiemetic *

c) Sedative and hypnotic *

d) Antiallergic *

9. Indications for the use of immunosuppressants:

a) Organ and tissue transplantation *

b) Tumor diseases

c) Autoimmune diseases *

d) Infectious diseases

10. Immunosuppressants include:

a) Prednisolone *

b) Histaglobulin

c) Azathioprine *

d) Cyclosporine *

11. Urodane is used for gout because:

a) Its components piperazine and lithium benzoate increase the solubility of uric acid *

b) Inhibits the absorption of uric acid in the renal tubules

c) Urodan can be taken in long courses *

12. Side effects of immunosuppressants are:

a) Immunodeficiency *

b) Arterial hypertension

c) Allergic reactions

13. The mechanism of anti-inflammatory action of NSAIDs is associated with:

a) Inhibition of phospholipase A 2

b) Reduced capillary permeability *

c) Inhibition of COX*

14. Second generation antihistamines include:

a) Fenistil *

b) Pipolfen

c) Diazolin

15. Immunocompetent cells that are the target of cyclosporine A:

a) B lymphocytes and macrophages

b) Selectively T-lymphocytes *

c) Selectively B-lymphocytes

d) T-lymphocytes and B-lymphocytes

16. Antiallergic drug that prevents the entry of calcium ions into mast cells:

a) Diphenhydramine

b) Ketotifen *

c) Loratadine

d) Tavegil

17. What side effect of acetylsalicylic acid distinguishes this drug from other non-steroidal anti-inflammatory drugs?

a) Bronchospasm

b) Hemorrhagic syndrome

c) Reye's syndrome *

d) Irritation of the gastric mucosa

e) Allergic reaction

18. Indications for the use of acetylsalicylic acid are:

a) Coronary heart disease *

b) Gastric ulcer

c) Fever in children with viral infection

19. A drug belonging to the group of selective COX 2 inhibitors:

a) Prednisolone

b) Diclofenac sodium

c) Celecoxib *

20. Specify the pharmacological effects of non-steroidal anti-inflammatory drugs:

a) Antipyretic and anti-inflammatory *

b) Analgesic and antiallergic

c) Anti-inflammatory and antiemetic

@Antibiotics 1

1. Specify the basic principles of antibiotic therapy:

a) Choosing the optimal dose of antibiotic *

b) Choosing the optimal route of antibiotic administration *

c) Prescribing minimally effective doses of antibiotics to reduce toxicity

2. Potentially toxic antibiotics include:

a) Penicillins

b) Levomycetin *

c) Aminoglycosides *

d) Tetracyclines *

3. The choice of antibiotics depends on:

a) Clinical diagnosis *

b) The most likely pathogen *

c) Spectrum of antibacterial activity of the antibiotic *

4. III generation penicillins are characterized by:

a) Activity against staphylococci

b) Activity against bacteria producing beta-lactamase

c) Activity against gram-negative bacteria *

5. Indicate the correct statements:

a) Bicillin is used for the same indications as penicillin

b) Bicillin is used for seasonal prevention of rheumatism *

c) Bicillin-5 is administered once a month *

6. Antistaphylococcal penicillins include:

a) Oxacillin *

b) Mezlocillin

c) Amoxiclav

7. Natural penicillin acts on the following cocci:

a) Streptococci *

b) Meningococci *

c) Pneumococci *

d) Gonococci *

8. Broad-spectrum penicillins include:

a) Ureidopenicillins *

b) Natural penicillins

c) Aminopenicillins *

d) Carboxypenicillins *

e) Protected penicillins *

9. Toxic effects of antibiotics are associated with:

a) Exceeding the dose of the drug *

b) Development of dysbacteriosis

c) Allergy of the patient

10. The combination of antibiotics gives the following effects:

a) Expanding the spectrum of antibacterial action *

b) Strengthening the antimicrobial effect *

c) Reducing the likelihood of developing bacterial resistance *

11. Ureidopenicillins include:

a) Ticarcillin

b) Piperacillin *

c) Amoxicillin

12. Long-acting penicillins include:

a) Phenoxymethylpenicillin

b) Bicillin-1 *

c) Bicillin-5 *

d) Novocaine salt of benzylpenicillin *

13. Carboxypenicillins include:

a) Ampicillin

b) Ticarcillin *

c) Azlocillin

d) Carbenicillin *

14. Typical indications for benzylpenicillin include:

a) Syphilis *

b) Meningococcal meningitis *

c) Erysipelas *

15. Indicate the correct statements:

a) Natural penicillins are broad-spectrum antibiotics

b) Protected penicillins are broad-spectrum antibiotics *

c) III generation penicillins are narrow-spectrum antibiotics

16. Antibiotics combined with beta-lactamase inhibitors include

a) Amoxiclav *

b) Ticarcillin

c) Unasin *

17. Indications for combined treatment with antibiotics:

a) Moderate infection

b) Severe infection *

c) Polyinfection *

18. Indicate one of the mechanisms causing bacterial resistance to β-lactam antibiotics:

a) Blockade of DNA gyrase

b) Translocase blockade

c) Enzyme inactivation *

@Antibiotics 2

1. Toxic effects of aminoglycosides include:

a) Vestibular disorders *

b) Hearing loss *

c) Kidney damage *

2. Adverse effects of lincomycin include:

a) Dysbacteriosis *

b) Pseudomembranous enterocolitis *

c) Kidney disorders

3. Toxic effects of tetracyclines include:

a) Nephrotoxicity *

b) Hepatotoxicity *

c) Damage to bone tissue *

4. Macrolides include:

a) Erythromycin *

b) Azithromycin *

c) Roxithromycin *

d) Clarithromycin *

5. Which of the following effects can be observed when a combination of streptomycin and gentamicin is simultaneously administered into the patient’s body:

a) Strengthening the antimicrobial effect

b) Increased ototoxicity *

c) Increased nephrotoxicity *

6. Indicate the correct statements:

a) The duration of treatment with aminoglycosides is limited to 7 - 8 days *

b) The dose of aminoglycosides varies widely depending on the severity of the infection

c) Aminoglycosides are not used simultaneously with loop diuretics *

7. The third generation of aminoglycosides includes:

a) Brulomycin *

b) Amikacin *

c) Netilmicin *

8. Indications for the use of macrolides are:

a) Infections of the upper and lower respiratory tract *

b) Lyme disease *

c) Infection of ENT organs *

d) Peptic ulcer of the stomach and duodenum *

9. Specify the correct statements regarding doxycycline:

a) Belongs to the group of penicillins

b) Can be prescribed once a day *

c) Has a wide spectrum of action *

10. Levomycetin is characterized by all of the following complications, except:

a) Blood lesions

b) Dermatitis

c) Acute productive psychosis

d) Myocarditis

e) Lesions of bone tissue *

11. The first generation of aminoglycosides includes:

a) Gentamicin

b) Streptomycin *

c) Kanamycin *

12. Which antibiotic is active against atypical pathogens (mycoplasma, chlamydia, legionella):

a) Gentamicin

b) Erythromycin *

c) Ampiox

d) Penicillin

13. Which antibiotics are safest to use during pregnancy:

a) Aminoglycosides

b) Tetracyclines

c) Levomycin

d) Penicillins *

14. For the treatment of pseudomembranous enterocolitis the following is used:

a) Doxycycline

b) Azithromycin

c) Vancomycin *

d) Imodium

e) Metronidazole *

15. Indicate the correct statements:

a) Levomycetin can be used in newborns with sepsis

b) Levomycetin is a broad-spectrum antibiotic *

c) Levomycetin is a low-toxic antibiotic

16. The antibacterial spectrum of “new” macrolides includes:

a) Gram “+” cocci*

b) Chlamydia *

c) Legionella *

17. Indicate the correct statement regarding azithromycin:

a) It is a narrow-spectrum antibiotic

b) Prescribed 3 times a day

c) Refers to macrolides *

18. Lincomycin has the following spectrum of action:

a) Gram”+” cocci *

b) Gram”-” bacteria

c) Pseudomonas aeruginosa

19. Specify an antibiotic that is contraindicated in children under 8 years of age:

a) Penicillin

b) Cefotaxime

c) Erythromycin

d) Tetracycline *

20. Specify the second generation macrolide:

a) Erythromycin

b) Doxycycline

c) Clarithromycin *

21. With the simultaneous introduction of a combination of streptomycin and gentamicin into the patient’s body, the following may develop:

a) Photosensitivity

b) Ototoxicity *

c) Toxic effects on bone tissue

d) Cholestasis

22. Select a drug from the group of aminoglycosides:

a) rifampicin

b) gentamicin*

c) erythromycin

23. Select a bacteriostatic drug:

a) erythromycin*

b) cephalexin

c) penicillin

24. Select a group of antibiotics with severe nephrotoxicity:

a) aminoglycosides*

b) penicillins

c) macrolides

25. Antibiotics are not indicated for:

a) viral infections*

b) coccal infections

d) minor surgical interventions

@ Antiviral drugs

1. Which anthelmintic drugs do not require diet when using them:

a) Fenasal

b) Levamisole *

c) Piperazine *

2. Antimalarial drugs include the following drugs:

a) Metronidazole

b) Primaquine *

c) Hingamin *

d) Nystatin

3. Metronidazole is used in the treatment of:

a) Giardiasis *

b) Trichomoniasis *

c) Amoebiasis *

d) Anaerobic infection *

4. Specify a drug that inhibits the synthesis of viral structural proteins and thereby disrupts the assembly of smallpox virions:

a) Azidotimidine

b) Acyclovir

c) Metisazon *

d) Midantan

5. Antiamoebic drugs include:

a) Midantan

b) Emetine *

c) Metronidazole *

6. Antifungal agents include:

a) Tetracycline

b) Terbinafine *

c) Amphotericin *

d) Nystatin *

7. The causes of fungal infections can be:

a) Use of sulfonamides combined with trimethoprim *

b) Use of amphotericin group antibiotics

c) Use of antibiotics of the penicillin group *

8. When treating nematodes, the following are used:

a) Male fern extract

b) Piperazine adipate *

c) Levamisole *

9. To prevent influenza, the following antiviral drugs are used:

a) Interferon *

b) Oksolin *

c) Remantadine *

10. Name a drug that blocks the reverse transcriptase of oncornoviruses and is used in the complex treatment of AIDS:

a) Acyclovir

b) Azidotimidine *

c) Midantan

11. Side effects of quinine include:

a) Bradycardia *

b) Dizziness *

c) Stimulation of uterine contractions *

12. Antimycotic agents include:

a) Clotrimazole *

b) Griseofulvin *

c) Nystatin *

13. When treating giardiasis, the following is used:

a) Metronidazole *

b) Tetracycline

14. Fenasal is used in the treatment of:

a) Ascariasis

b) Hymenolepiasis *

c) Enterobiasis

15. Antiviral drugs include:

a) Metronidazole

b) Acyclovir *

c) Nystatin

16. Indicate the correct statements:

a) Terbinafine has a fungicidal effect *

b) Terbinafine is effective against fungal infections of nails *

c) The average dose of terbinafine when taken orally is 1000 mg

17. Indicate the correct statements:

a) Remantadine is effective for the treatment of influenza A *

b) Remantadine is effective for the treatment of influenza B and C

c) Remantadine is effective when given early *

18. Indicate the correct statements:

a) Metronidazole is used in the treatment of anaerobic infection *

b) Metronidazole is used to treat protozoal infections *

c) Metronidazole is effective against Pseudomonas aeruginosa

19. Specify a drug used to prevent influenza:

a) Azidotimidine

b) Idoxuridine

c) Acyclovir

d) Rimantadine *

@Anti-tuberculosis drugs

1. Prevention of neuritis caused by isoniazid includes:

a) Vitamin D

b) Vitamins B 1 and B 6 *

c) Vitamin C

2. Intestinal quinolones act on:

a) Gram "-" bacteria *

b) Amoebas *

c) Gram "+" bacteria

3. Systemic quinolones include:

a) Ofloxacin *

b) Pefloxacin *

c) Ciprofloxacin *

d) Levofloxacin *

4. Urinary quinolones include:

a) Nevigramon *

b) Ofloxacin

6. Specify the measure used to prevent crystalluria caused by sulfa drugs (SAP):

a) Taking SAP before meals

b) Combination of taking SAP with plenty of fluids and administration of Diacarb *

c) Combination of SAP with ascorbic acid

7. General principles of chemotherapy for tuberculosis:

a) Combination of 2-3 anti-tuberculosis drugs *

b) Long-term course of treatment *

c) Taking into account the intracellular localization of mycobacteria *

8. Specify an anti-tuberculosis drug that inhibits the synthesis of mycolic acids and, therefore, disrupts the structure of the cell wall of Mycobacterium tuberculosis:

a) Rifampicin

b) Streptomycin

c) Isoniazid *

9. Side effects of sulfonamides include:

a) Hematotoxicity *

b) Crystalluria *

c) Damage to bone tissue

10. Indications for the use of intestinal quinolones are:

a) Enterocolitis *

b) Dysentery *

c) Putrefactive fermentation *

11. Indicate the correct statement:

a) Sulfonamides have a bactericidal effect

b) Sulfonamides have a bacteriostatic effect *

c) Salazosulfonamides are compounds of sulfonamide with trimethoprim

12. Sulfonamides that are poorly absorbed when taken orally include:

a) Sulfalene

b) Phthalazol *

c) Sulgin *

13. Intestinal quinolone preparations are:

a) Intestopan *

b) Ofloxacin

c) Enteroseptol *

14. The antibacterial spectrum of sulfonamides includes:

a) Gram "+" bacteria *

b) Gram"-"bacteria *

c) Chlamydia *

15. Indicate the correct statement:

a) Trimethoprim interferes with the formation of folic acid by the microorganism

b) The combination of trimethoprim with sulfanilamide gives a bacteriostatic effect

c) Co-trimoxazole is contraindicated during pregnancy and nursing mothers *

16. Easily absorbed sulfonamides include:

a) Streptocide *

b) Phthalazol

c) Sulgin

17. The drug combined with trimethoprim is:

a) Biseptol *

b) Sulgin

c) Clotrimazole

18. Ofloxacin:

a) Has a bactericidal effect*

b) Disturbs the process of DNA helixation by blocking DNA gyrase*

c) Disrupts cell wall synthesis

19. The mechanism of antimicrobial action of fluoroquinolones:

a) Inhibition of DNA polymerase and disruption of DNA synthesis

b) Inhibition of DNA gyrase and disruption of DNA supercoiling *

c) Inhibition of transpeptidase and disruption of cell wall synthesis

CATEGORIES

POPULAR ARTICLES

2023 “kingad.ru” - ultrasound examination of human organs